The Flat Earth Society

Flat Earth Discussion Boards => Flat Earth Investigations => Topic started by: ICanScienceThat on May 20, 2019, 11:19:44 PM

Title: Problems with the Heliocentric Model
Post by: ICanScienceThat on May 20, 2019, 11:19:44 PM
Last summer, I had a thread where I asked for genuine questions about how things work with the standard mainstream model of RET. I'd like to open that topic up once more.

Is there something about garden-variety RET that seems wrong? Something that was never adequately explained? I'd love to help you science anything.

Motion of Sun, Moon, & Stars? Phases of the Moon? Eclipses? Does the horizon rise to eye level? How does the curve calculator work? Does the curve calculator work? What's a gyrocompass? What the heck is the "spacetime metric?" Anything you like.
Title: Re: Problems with the Heliocentric Model
Post by: Tim Alphabeaver on May 21, 2019, 08:05:11 AM
Since you mentioned it, what the heck is a spacetime metric?
Title: Re: Problems with the Heliocentric Model
Post by: ICanScienceThat on May 21, 2019, 10:35:00 PM
Since you mentioned it, what the heck is a spacetime metric?
First of all, nice chinchilla.   :)

The full explanation of the spacetime metric is VERY mathy. Let me try to summarize a bit.
The full form of Einstein's general relativity equation looks like... erm I have no idea how to do equations in this...
So it's this equation with something typically written as "g sub uv" on the left and "T sub uv" on the right.
We call the "T sub uv" the "stress tensor" and the "g sub uv" is the "spacetime metric" or just the "metric".

What the what now?

Very loosely, that stuff on the right-hand side represents the mass and energy in the system. The stuff on the left describes the movement of objects in space and time. Put very simply, if you can count up all the energy in a region of space and put it on the right-hand side of that equation, the left-hand side will tell you how everything in that region should move.

Going just a little deeper, the "spacetime metric" part of the equation describes the shape of space and time... and once again the what now?

There are 2 ways of looking at this...

1) The spacetime metric describes how objects move. We understand that objects in a gravity well are pulled towards the center of that gravity. This equation describes how the object's position in the future is affected by gravity.
2) The spacetime metric describes how space and time are actually curved so that an object moving in what appears to be a straight line from the object's perspective is curved to a perspective somewhere else.

That number 2 there is bizarre.  I'm about to dig a little deeper into the math, so just quit reading wherever you get bored...

What's a "vector"? Let's describe motion in 2D. We'll call the direction from West to East the "x axis". We'll call South to North the "y axis". You can describe your position and velocity as some combination of x and y. So bundle the x and the y together and call them a "vector." Your velocity can be thought of as a single value v. We understand that v has an x component and a y component, and together v.x and v.y are collectively known as v. Simple. Add altitude and we have a 3D vector. There you go... 3D space.

How does time fit in? At some time t=t0, you were at location p0 (a vector in 3D space). At some later time t=t1, you had moved to position p1. We could add t to our bundle of data to make the 4D vector (p.x, p.y, p.z, p.t). Now we can put that together like so: (p0.x, p0.y, p0.z, t0) or maybe we'd just call that (p0.x, p0,y, p0.z, p0.t)... collectively p0 as a 4D vector.

Why? Honestly, let's just start with "Why not?" We can do this if we want, so we did it. Justification not needed. It's a mathematical abstraction.

We now have a 4D coordinate space... "spacetime". Space and time combined into a single mathematical abstraction. Simple as that.

Einstein was struggling with how to resolve relativity (now known as "special relativity") and gravity. He started with what we call the "equivalence principle." This states that acceleration due to gravity feels EXACTLY like the inertial force you feel under acceleration. Putting that another way, free-fall feels EXACTLY like zero-g.

When he combined this with the Lorentz transforms of special relativity, this didn't make any sense. How can light not tell the difference between an accelerating frame and a gravity well?

Through a lot of hard work and some pretty heavy math, he eventually realized that if you describe the world using 4D coordinates "spacetime", AND you allow for the "spacetime" to be curved arbitrarily, the paradox can be resolved.

Einstein described the equations of motion using 4D spacetime. The radical part comes in when we allow for what we consider to be the "x direction" (remember that was East) to vary. At any given place and time, x is fixed pointing East. But what if at some other location or time, x could point in some other direction? Remember how we said our 4D axis was (East, North, Altitude, Time)? To picture this, imagine your East, North, Altitude axes on a globe (or on an AE projection if you prefer). The direction of "East" is different at different locations. If you keep going East, you'll eventually get back where you started from. Einstein's equations allowed for this exact situation. He started with an arbitrary 4D coordinate system to describe "spacetime". He didn't decide up front that x must point East. Instead he solved the equations and allowed the equations to tell HIM which way the x axis pointed.

Does that make any sense? Einstein abstracted away the coordinate system in which he described the forces and motions. He constrained this equation with what we knew about the physical universe and used that to solve for what the coordinate system looked like. THAT is the "spacetime metric". That's what the coordinate system looks like based on the way the universe behaves.

What'd we discover? What we see is that the presence of a point-mass on the right-hand side of the equation (a high density energy), the coordinate system becomes curved. That really shouldn't sound all that shocking at this point. After all, we invented our coordinate system with North, East, and Up in the first place right? And we know that ended up being curved, so why shouldn't the "spacetime metric" end up curved too?

What's really interesting about this curved spacetime metric is what happens if we place a small, stationary object in space near a point mass. If we solve for the future location of our stationary object, we see that it will have moved towards the point mass. If we put in a small value for the point mass, the solution for the future location of our object precisely matches Newton's universal law of gravitation. Only this time, we don't get there using F=mA. We get there just by solving the 4D coordinate system. Neat right?

So what? Well, what if instead of a stationary object, we put a beam of light next to our point mass? Prior to 1915, we had no reason to expect that gravity affected light, but this new "spacetime metric" gave the same solution for light that it did for matter - the light should bend towards the point mass. This was an unexpected prediction made by this new equation, and in 1919 that prediction was confirmed to be accurate. We now know that light is affected by gravity.

There are several other very surprising results that fall out of this equation, and one by one, each one has been confirmed. We know this equation must still be incomplete in some small way, and theoretical physicists and mathematicians are in a race to the next major breakthrough.

Summary: The "spacetime metric" is the part of Einstein's field equations (general relativity) that describe how an object moves through space. We usually describe this as time and space being curved, but to be precise, we're really talking about the motion of an object that is curved in space and time. Is it the motion that's curved or spacetime that's curved? That's sort of a philosophical question really. We can be certain that the math says the thing's gonna curve, and on that we can all agree.

But is this all theoretical? I mean, it's just math right? Many times, the math can give us new insights into the behavior of the physical world, and this is the perfect example. The math said that light should bend, and guess what... now that we knew to look for it, we were able to show that light DOES bend. The math says time passes differently in the presence of gravity, and guess what... it does! We don't just take the math and say, "oh well... math says it's true so I guess we'll just accept it." No, we look at the math and say, "according to this math, this should be true... let's see if we can test that!" The math has taught us a lot about the realities of the physical world... it shows us what to look for.
Title: Re: Problems with the Heliocentric Model
Post by: Auditor on May 22, 2019, 07:59:00 AM
Hmm, well yes of course.
Title: Re: Problems with the Heliocentric Model
Post by: ICanScienceThat on May 23, 2019, 10:41:09 PM
The "spacetime metric" is a pretty complex topic. The super-short version is, "Einstein figured out that space and time are related, and oh, by the way, they are both curved." But naturally, that simple statement earns nothing but scoffs from all over the science spectrum.

A better description is, "Einstein did some math that said time and space can bend. We checked it, and it turns out his equations work really well to describe reality." But that still leaves most readers pondering what the hell bent space or bent time even means.

So I prefer, "Einstein did some math. In his equations, we have a term we call the 'spacetime metric.' This thing describes the way stuff moves through space. The equations predict what we all know of as gravity."

Is that any better?

Any other requests out there?
Title: Re: Problems with the Heliocentric Model
Post by: Auditor on May 24, 2019, 09:53:46 AM
No, I thought your spacetime metric explanation was real good. Not to say I understand it 100%. I have an ok grasp of it though.
Title: Re: Problems with the Heliocentric Model
Post by: Tom Bishop on May 25, 2019, 03:51:25 AM
There are no "problems" with the heliocentric model because it is composed of hundreds of years of a concentrated effort to construct confutations to explain why things are not as they seem.

Any motionless earth experiments are explained with length shortening, changes to the understanding of space and time, anything it takes.

A situation where the most sensitive experiments created by science are unable to detect the gravitational influence of the sun or moon? Easily explained by selective gravity; "preferred curves" in space-time, which the test bodies in the experiments are following.
Title: Re: Problems with the Heliocentric Model
Post by: ICanScienceThat on May 25, 2019, 03:58:15 AM
There are no "problems" with the heliocentric model because it is composed of hundreds of years of a concentrated effort to construct confutations to explain why things are not as they seem.

Any motionless earth experiments are explained with length shortening, changes to the understanding of space and time, anything it takes.

A situation where the most sensitive experiments created by science are unable to detect the gravitational influence of the sun or moon? Easily explained by selective gravity; "preferred curves" in space-time, which the test bodies in the experiments are following.

Tom, is there anything in there that you'd like me to address? My assessment is that this post is not a request for any explanations.

Happy to address any genuine questions. No matter who they come from. Not going to argue anything, but if anyone wants an explanation, I'll do my best.
Title: Re: Problems with the Heliocentric Model
Post by: iamcpc on May 28, 2019, 04:53:00 PM
There are no "problems" with the heliocentric model because it is composed of hundreds of years of a concentrated effort to construct confutations to explain why things are not as they seem.


Seriously Tom? If there are no problems with that model then why is the community so fragmented with dozens, if not hundreds of different models?

I'll give you a hint:

Some models do very well at explaining some things while other models do well at explaining others. Every model has problems that's why alternate models are being made.
Title: Re: Problems with the Heliocentric Model
Post by: AATW on June 05, 2019, 03:30:05 PM
There are no "problems" with the heliocentric model because it is composed of hundreds of years of a concentrated effort to construct confutations to explain why things are not as they seem.
That's somewhat of a misrepresentation, Tom.
Scientific models simply get updated or replaced as we learn more.

For centuries the geocentric model ruled the roost. It does a very good job of explaining the path of the sun and stars, it does look like we're at the centre of things. It wasn't till 1534 that Copernicus published his book advocating the heliocentric model based on the way the planets move and it took quite a long time for that to be accepted. For quite a while attempts were made to make the geocentric model work, so ingrained was it. Ultimately though, the heliocentric model just works better and more accurately matches observations.

Similarly, Newton's model of gravity and motion was thought to be the definitive model for a long time, some of the findings a century or so ago didn't work well with that model and that's when Relativity entered the fray. That's more of an amendment to Newton than a complete replacement in that at "normal" speeds the equations reduce to Newton's, but it does more completely explain observations.

This is how science work. It's weird that you sneer at this and regard it all as fudges when your model has lots of things like this - the sun's consistent angular size is explained by a fudge where randomly some objects maintain the same size regardless of distance. There are lots of examples in your model. And you're very scathing of science in general but cherry pick bits of it like Special Relativity to explain why in UA we don't go past the speed of light. I'm not sure why the speed of light needs to be a universal speed limit in your model.

Most of your problems with the heliocentric model are you just not understanding it properly, like the recent one about shadow direction in an eclipse. And admittedly that does take some thinking about but if you take the trouble to you'll see that these problems don't really exist.
Title: Re: Problems with the Heliocentric Model
Post by: somerled on July 25, 2019, 05:56:00 PM
The "spacetime metric" is a pretty complex topic. The super-short version is, "Einstein figured out that space and time are related, and oh, by the way, they are both curved." But naturally, that simple statement earns nothing but scoffs from all over the science spectrum.

A better description is, "Einstein did some math that said time and space can bend. We checked it, and it turns out his equations work really well to describe reality." But that still leaves most readers pondering what the hell bent space or bent time even means.

So I prefer, "Einstein did some math. In his equations, we have a term we call the 'spacetime metric.' This thing describes the way stuff moves through space. The equations predict what we all know of as gravity."

Is that any better?

Any other requests out there?

I always thought that Mrs Einstein did the the math since Albie wasn't very good in that department . Lorentz transformation was lifted straight from aether based relativity - which transformation occured with respect to the aether.
 
I have a 1921 physics book ( authors are associates of the royal college of science) in which it is stated light , as radiation , propagates through the medium aether. So what happened in 1919 that allowed Einsteinian relativity to be described as proven when the aether medium was still taught in1921?
Title: Re: Problems with the Heliocentric Model
Post by: dirtysnowball on July 25, 2019, 07:31:45 PM
Quote
So what happened in 1919 that allowed Einsteinian relativity to be described as proven when the aether medium was still taught in1921

Flat Earthers claim that most of the mainstream science that is being taught in the 21st century comes as a result of 'lies' and a way of covering up the truth.  So your example above could be construed by some as another example of a lie.
Title: Re: Problems with the Heliocentric Model
Post by: markjo on July 25, 2019, 07:49:23 PM
So what happened in 1919 that allowed Einsteinian relativity to be described as proven when the aether medium was still taught in1921?
In 1919, Arthur Eddington took some photos of a total solar eclipse and showed that the background stars had exhibited signs of gravitational lensing as predicted by general relativity.  It usually takes a while for latest scientific theories to make it into the text books and school curriculums.
Title: Re: Problems with the Heliocentric Model
Post by: Tom Bishop on July 25, 2019, 08:32:05 PM
I recently added some information on aether here, to the 'aether' page: https://wiki.tfes.org/Aether
Title: Re: Problems with the Heliocentric Model
Post by: somerled on July 25, 2019, 08:42:55 PM
Eddington captured 16 images of the eclipse at Principe . One was usable and gave a value of displacement of 1.61 arc seconds - Einsteins theory predicted 1.75 .

The team sent by the Astronomer Royal to Sobral in Brazil obtained several images but after developing 4 plates admitted that nothing could be gained from these but obtained a value for deflection ( of Mercury ) of 0.9 arc seconds .

Read the whole sorry saga in the book " The Secret Anarchy of Science " , by Michael Brookes who is not a FEr os renegade . You'll see why it takes so long for "acceptance " . These things are brought in through the back door when nobody is looking so to speak . Same goes for the Heliocentric model . Nothing new brought to the table - no new evidence .
 
The point about Mrs Einstein still stands .

Thanks for posting that aether info Tom Bishop , will have a look at that .
Title: Re: Problems with the Heliocentric Model
Post by: Tom Bishop on July 27, 2019, 01:38:58 AM
Thank you for the reference. I was planning on making an article about the starlight deflection at some point.

My opinion is that relativity and much else is adhoc-ery to hide that Heliocentrism was shown to be false by experiment.
Title: Re: Problems with the Heliocentric Model
Post by: Zonk on July 28, 2019, 08:58:42 PM
Flat Earthers claim that most of the mainstream science that is being taught in the 21st century comes as a result of 'lies' and a way of covering up the truth.  So your example above could be construed by some as another example of a lie.

2 questions immediately come to mind.

1:  Why?  Why put forth so much effort, creating a conspiracy involving  millions of people, to hide such a benign "truth"?

2:  How has there never been one person involved in the conspiracy to come forth and spill the beans? 

The first is exceedingly implausible.  The second is obviously impossible. 
Title: Re: Problems with the Heliocentric Model
Post by: somerled on July 29, 2019, 08:03:08 AM
1. Control/enslavement of the humanity . The conspiracy does not require the involvement of millions , merely requires millions to believe anything the authorities tell them to believe

2. There have always been dissenters , scientists and others.

Visit the link Tom posted . Here's another https://www.reddit.com/r/Geocentrism/comments/2oz7ye/quotes_from_famous_scientists_on_geocentrism/
 
Accepted mainstream models are not proven models . The peer review system is used to promote science within the accepted model and discourages critical thinking and can thus ignore experimental proof that the mainstream model is wrong .
Title: Re: Problems with the Heliocentric Model
Post by: stack on July 29, 2019, 08:40:31 AM
1. Control/enslavement of the humanity . The conspiracy does not require the involvement of millions , merely requires millions to believe anything the authorities tell them to believe

Control/enslavement of humanity to what end? How is 'flat earth' breaking the chains that control and enslave us? How am I set free by a flat earth? It would make little to no difference to me if I woke up tomorrow and all the textbooks had been changed to a flat earth or a geocentrism centric model.
Flip the script, guaranteed, if the mainstream considered the world flat, fringe groups would pop up claiming "you're all controlled/enslaved into this flat earth indoctrination spoon fed to you by the authority, blah, blah, blah..."
This 'control/enslavement' business is neither here nor there.
Title: Re: Problems with the Heliocentric Model
Post by: Zonk on July 29, 2019, 12:56:48 PM
Quote
1. Control/enslavement of the humanity . The conspiracy does not require the involvement of millions , merely requires millions to believe anything the authorities tell them to believe

It does actually.  For one small example from my world, every day, hundreds of people fly on Southern Hemisphere inter continental flights.  By your FE model, these distances are 2-3 times longer than what they actually are.  Longer than the published range of any current airliner.  Under your current model, one of more of the following has to be going on:

1:  The published ranges of the longest range aircraft are incorrect, and they actual are capable of such long range flight.  This would require the cooperation of airlines, the pilots themselves, the aircraft manufacturers, and air traffic control.  Also the cooperation of every passenger, who, upon being told it would be a 9 our flight, winds up on a 22 hour flight.  It's one thing to believe faked moon landings.  It's quite another to believe that the 22 hour flight was only 9.

1a:  The aircraft are not only capable of much longer range, but much higher speeds.  However, to hide this fact from the public, these higher speeds are used on only a select few southern hemisphere flights and not in the northern hemisphere where the vast majority of trans oceanic flights occur. And not a single person who knows about this capability, again, airline executives, pilots and mechanics, aircraft manufacturers, aeronautical engineers, etc, have spoken one word about it. 

2:  There are no super fast, super long range airliners, and all the published non-stops are a hoax, and every time one takes off, they are routed through a northern hemisphere city for a refueling stop and aircraft change, and not a single passenger has ever said a word about this. 

3:  On a flat earth model, unless one is going due north or due south, one has to continuously bank to the left or the right to stay on course.  I am a pilot and I have never once had to do this.  If your destination is at a heading of 060, you can pick that heading, level your wings, and eventually get there.  On a FE model, you have to continually correct left to maintain 060, or soon you will be at 065, 070, 075...etc.  If you fly long enough, soon you will be going south.  This phenomenon simply does not occur.  So now your conspiracy has to include not just souther hemisphere intercontinental pilots, but every single private, commercial, and military pilot worldwide.

None of that involves believing what authorities say.  It's willfully disbelieving your own experiences.
Title: Re: Problems with the Heliocentric Model
Post by: Tim Alphabeaver on July 29, 2019, 02:37:12 PM
The peer review system is used to promote science within the accepted model and discourages critical thinking and can thus ignore experimental proof that the mainstream model is wrong .
I strongly disagree with this. If you were right, then places like CERN wouldn't exist. They are explicitly searching for physics that is beyond the Standard Model.
Title: Re: Problems with the Heliocentric Model
Post by: Zonk on July 29, 2019, 02:43:22 PM
The peer review system is used to promote science within the accepted model and discourages critical thinking and can thus ignore experimental proof that the mainstream model is wrong .
I strongly disagree with this. If you were right, then places like CERN wouldn't exist. They are explicitly searching for physics that is beyond the Standard Model.

That's what these conspiracy theorists fail to understand, or if they do understand, are ignoring.  Scientists live to make new discoveries.  It's why they got into science in the first place.  Anyone who claims that scientists cover up the truth in order to preserve the status quo don't know the first thing about science and scientists.  Scientists don't get recognition, promotions, and grant money for merely confirming what is already know.  They achieve that by making new discoveries. 
Title: Re: Problems with the Heliocentric Model
Post by: newhorizons on July 30, 2019, 07:18:22 PM
I am not going to go into a huge amount of detail with this but I agree that it has never been proved that the heliocentric model of the solar system (Universe or whatever you want to call it) is correct. Indeed, if you limit your fact finding to what we directly experience through observation it seems ludicrous to think that the Earth is anything other than stationary at the centre of everything we can see (i.e. the Universe). The Earth it seems to us from any single vantage point upon its surface is a flat plane with the heavens rotating above us.

Our ancestors witnessed events happen in the skies above us without any input from them at all. Something was causing those events to happen though and in the absence of any other logical explanation they believed that some unseen power, force or entity was behind it all. This entity could be described now as 'God'. The supposition that the planets, Sun and Moon somehow orbit above the surface of the Earth was accepted for many a century. Some of the events observed could be accounted for by this model quite easily but others less so. Then through the earliest (recorded) telescopic observations in the early 17th century it was noted that Venus shows a phase cycle identical to the Moon. As telescopes became more refined it was noticed that Mercury also showed the same. None of the other known planets at the time, Mars Jupiter or Saturn showed this same pattern. These phases were difficult to account for if the Earth was at the assumed centre of the Solar System but were easily explained if you place the Sun at the centre instead and then assume that Mercury and Venus follow orbits that are nearer to the Sun than the Earth.  Not only does Venus show phases but it also grows in visible size as it turns into a crescent shape and dwindles in size as it approaches 'full'. These are observations that can be repeated today in any amateur telescope.

So while the heliocentric model has never been proved, there is very strong evidence to support it. Unlike I would suggest, the models proposed by modern FET.

Title: Re: Problems with the Heliocentric Model
Post by: somerled on July 31, 2019, 08:14:20 AM
I would like to point out that the planetary phases of Venus and Mercury were explained easily in the Tycho Brahe geocentric model . These planets orbited the sun which in turn orbited the earth .
        Heliocentrism was founded on no new evidence .
Title: Re: Problems with the Heliocentric Model
Post by: newhorizons on July 31, 2019, 09:56:16 AM
Tycho proposed that the Sun and Moon orbit the Earth while all other planets orbit around the Sun.

If this were true then why is it that only Mercury and Venus show a complete cycle? Surely if Mars, Jupiter and Saturn were orbiting around a Sun that was in turn orbitting around the Earth then from our vantage point n Earth we would see a cresent Mars, Jupiter and Saturn as well. Which we never do.
Title: Re: Problems with the Heliocentric Model
Post by: totallackey on July 31, 2019, 10:14:52 AM
That's what these conspiracy theorists fail to understand, or if they do understand, are ignoring.  Scientists live to make new discoveries.  It's why they got into science in the first place.  Anyone who claims that scientists cover up the truth in order to preserve the status quo don't know the first thing about science and scientists.  Scientists don't get recognition, promotions, and grant money for merely confirming what is already know.  They achieve that by making new discoveries.
I agree they don't get recognition for merely confirming what is already known...

The problem is they continue to get money for perpetuating the things we obviously don't know, a' la' melting ice sheets and climate modeling, rather than actual science.
Title: Re: Problems with the Heliocentric Model
Post by: somerled on July 31, 2019, 11:15:28 AM
Tycho proposed that the Sun and Moon orbit the Earth while all other planets orbit around the Sun.

If this were true then why is it that only Mercury and Venus show a complete cycle? Surely if Mars, Jupiter and Saturn were orbiting around a Sun that was in turn orbitting around the Earth then from our vantage point n Earth we would see a cresent Mars, Jupiter and Saturn as well. Which we never do.

This is the Brahe model I referred to

 http://www.polaris.iastate.edu/EveningStar/Unit2/unit2_sub3.htm

Mercury and Venus orbit the sun . In his model Earth is stationary - science is unable to provide proof of rotation . Apparently outer planets do exhibit phases although I've not looked into that yet .
Title: Re: Problems with the Heliocentric Model
Post by: newhorizons on July 31, 2019, 11:36:08 AM
The outer planets show a very slight flattening (gibbous) effect. Most pronounced in the case of Mars and least with Saturn. The outer planets never show a crescent phase. I know that because I have been observing the planets through telescopes for many years. So my evidence for that is direct observation.

The current model for the formation of solar system is that it formed from an accretion disk of gas and dust that surrounded a protostar that we now called the Sun.

This model is supported by our modern day observations of other solar systems forming around other stars. This stellar centric solar system model certainly accounts for all the observations we make of the planets and is I would suggest rather more simple and logical compared to Tychos models. In fact I can see several problems with Tychos model. For example it allows for an angle between Saturn the Earth and Sun would produce an almost half phase (1st quarter) Saturn which in the real world (as seen from Earth) never happens. Photos of a 'half Saturn' have been taken but only from spaceprobes like Cassini.

You have used that word proof again rather than evidence when it comes to favouring one model over another. Tycho assumed the Earth was stationary because he could not feel it moving. That is the same for us as it was for him but with the benefit of the further progress in science that has been made since Tychos day we now know why we have no direct sensation of the Earths orbit around the Sun. We see the evidence rather than feel it.
Title: Re: Problems with the Heliocentric Model
Post by: somerled on July 31, 2019, 03:54:43 PM
Science is unable to provide proof . This is the correct use of the term. Several experiments failed to find rotation . Strong evidence of a stationary earth  . We feel that and we see the sun cross the sky .
Title: Re: Problems with the Heliocentric Model
Post by: Zonk on July 31, 2019, 04:10:31 PM
Science is unable to provide proof . This is the correct use of the term. Several experiments failed to find rotation . Strong evidence of a stationary earth  . We feel that and we see the sun cross the sky .

Can you feel yourself going 500 MPH in an airliner at cruise?   No, you can't.  By your standard of evidence, the airplane isn't moving.  The earth is moving below you.  Why?  Because you can't feel the speed and you see the earth moving.
Title: Re: Problems with the Heliocentric Model
Post by: newhorizons on July 31, 2019, 05:25:15 PM
Quote
Science is unable to provide proof . This is the correct use of the term. Several experiments failed to find rotation

I am more curious to know what you define as 'stationary'.  Evidence of the Earth rotating and orbiting the Sun lies not in what you feel but what you experience. Science doesn't and has never set out to 'prove' anything, It merely presents evidence.  It is then left to us as individuals whether to accept or dismiss the evidence.  The Earths rotation causes day and night and we use the Earths orbit to define a time period that we call a year. Finally the tilt of the Earths axis, combined with its motion around the Sun result in the seasons. Your interpretation or belief (call it what you like) as a flat Earther is different and of course you are entitled to think that way if you wish.


Going back on our earlier discussion, Tycho was a highly respected observer and used some very accurate instrumentation for his time. His attention to detail in terms of logging the positions of the planets was immense and his records proved invaluable to helping Kepler develop his laws of planetary motion.  The combined efforts of Tycho and Kepler was a beautiful example of the scientific method in action. As a man of the church, Kepler believed passionately that the planetary orbits should be circular but he couldn't get his predicted planetary positions to match Keplers observed positions. Only by acting contrary to his own beliefs and making the planetary orbits elliptical instead could he finally get his predictions to agree with Keplers records exactly.

Despite being an excellent observer, Tychos ideas about the layout of the solar system though left a lot to be desired. The model that he developed with the Sun orbiting the Earth cannot possibly account for everything we experience. One major flaw with the diagram that you sent the link to puts the Sun on a potential collision path with Mars for example!  Surely even you must accept that is not possible?!?
Title: Re: Problems with the Heliocentric Model
Post by: somerled on August 01, 2019, 06:23:58 AM
I already know the heliocentric theory - it's thrust upon us when we are young and impressionable . It's brainwashing and leads to unquestioning acceptance of authority.

The term stationary is self evident . It is what we experience about our earth - we do not feel any rotation .

Rose tinted spectacles have no place in scientific endeavour .

Brahe's model explained everything he observed completely so what was this " lot to be desired ".

Brahe was given an assistant , Kepler , that he did not want or require . He actually threw him out . Again Kepler was manouvered  back into Brahe's employment and shortly after Brahe was dead , some say murdered by Kepler .

Kepler proceeded to steal all Brahe's work . The courts of King Frederick ordered the return of this body of work to it's rightful place . Kepler returned all but the data on the planet Mars - which disappeared .

Kepler's laws are Kepler's Fantasy , they do not exist. Where is the derivation of these laws . Laws are exact . Planetary orbits supposedly loosely fit these laws .

That version of Brahe's geocentric model which I linked was obviously drawn by a cloth headed heliocentrist  - he put's the sun at the centre of his representation of Brahe's geocentric model haha. Should be easy for you to look up the real one , if you are interested .






Title: Re: Problems with the Heliocentric Model
Post by: newhorizons on August 01, 2019, 06:50:24 AM
Quote
That version of Brahe's geocentric model which I linked was obviously drawn by a cloth headed heliocentrist

If you are so dismissive of that version of Brahes model then why did you send the link in the first place?

You don't need to give me a history lesson on Tycho or Kepler. I have a degree in astronomy so I am very familiar with the background.  What I would like you to do is to bring some new compelling evidence to the discussion that makes me think 'Hang on he may be right..' So far you haven't don't that. All you have done is make scathing comments that are quite typical of a conspiracy theorist about something you don't want to believe in.
Title: Re: Problems with the Heliocentric Model
Post by: somerled on August 01, 2019, 07:37:37 AM
You displayed a lack of knowledge about the background which is why I did have to give you a lesson on Tycho and Kepler and the geocentric model . You showed this - post 24 on this thread .

Those are not scathing comments on Kepler - they are truthful - there is no experimental/observational derivation of his supposed laws .

What compelling evidence exists for this heliocentric model ?
Title: Re: Problems with the Heliocentric Model
Post by: totallackey on August 01, 2019, 10:34:35 AM
Science is unable to provide proof . This is the correct use of the term. Several experiments failed to find rotation . Strong evidence of a stationary earth  . We feel that and we see the sun cross the sky .

Can you feel yourself going 500 MPH in an airliner at cruise?   No, you can't.  By your standard of evidence, the airplane isn't moving.  The earth is moving below you.  Why?  Because you can't feel the speed and you see the earth moving.
Your reply details an indoor/outdoor comparison and is invalid.
Title: Re: Problems with the Heliocentric Model
Post by: Zonk on August 01, 2019, 01:44:19 PM
Science is unable to provide proof . This is the correct use of the term. Several experiments failed to find rotation . Strong evidence of a stationary earth  . We feel that and we see the sun cross the sky .

Can you feel yourself going 500 MPH in an airliner at cruise?   No, you can't.  By your standard of evidence, the airplane isn't moving.  The earth is moving below you.  Why?  Because you can't feel the speed and you see the earth moving.
Your reply details an indoor/outdoor comparison and is invalid.

If earth is surrounded by a dome, isn't everything indoors?

In any event, what does that have to do with anything?  One cannot feel stable, non-accelerating motion whether one is on a rotating globe or in a vehicle.  Even if you are on a bicycle, you can't feel the motion.  You can feel the wind on your face, but you can feel wind on your face standing still also.  But you can't feel the motion if you are not accelerating.  If you jump out of an airplane at a high enough altitude to free fall for a while, you will initially feel the acceleration and the wind, two distinct feelings.  once you reach terminal velocity, you will no longer feel the acceleration and feel only the wind, the same feeling you would get if you were standing still in a wind tunnel.

If you were in space, moving at 100,000KPH, you would have no sensation of motion.  As you pass a stationary object, it would appear that you were motionless and the object were moving at 100,000 KPH in the opposite direction.
Title: Re: Problems with the Heliocentric Model
Post by: totallackey on August 01, 2019, 02:36:39 PM
Science is unable to provide proof . This is the correct use of the term. Several experiments failed to find rotation . Strong evidence of a stationary earth  . We feel that and we see the sun cross the sky .

Can you feel yourself going 500 MPH in an airliner at cruise?   No, you can't.  By your standard of evidence, the airplane isn't moving.  The earth is moving below you.  Why?  Because you can't feel the speed and you see the earth moving.
Your reply details an indoor/outdoor comparison and is invalid.

If earth is surrounded by a dome, isn't everything indoors?

In any event, what does that have to do with anything?  One cannot feel stable, non-accelerating motion whether one is on a rotating globe or in a vehicle.  Even if you are on a bicycle, you can't feel the motion.
As someone who has ridden a bike extensively, this is patently false. 
You can feel the wind on your face, but you can feel wind on your face standing still also.
And the perceived force of the wind increases as you move.
But you can't feel the motion if you are not accelerating.  If you jump out of an airplane at a high enough altitude to free fall for a while, you will initially feel the acceleration and the wind, two distinct feelings.
And here you contradict your own position within one sentence. 
once you reach terminal velocity, you will no longer feel the acceleration and feel only the wind, the same feeling you would get if you were standing still in a wind tunnel.
As you admit yourself, in the above statement, false.
If you were in space, moving at 100,000KPH, you would have no sensation of motion.  As you pass a stationary object, it would appear that you were motionless and the object were moving at 100,000 KPH in the opposite direction.
Wrong.

There are five senses.
Title: Re: Problems with the Heliocentric Model
Post by: ChrisTP on August 01, 2019, 03:17:52 PM
"And the perceived force of the wind increases." - Unless you're cycling in the direction of the wind then you feel like there is no wind at all sometimes.
Title: Re: Problems with the Heliocentric Model
Post by: totallackey on August 01, 2019, 03:42:36 PM
"And the perceived force of the wind increases." - Unless you're cycling in the direction of the wind then you feel like there is no wind at all sometimes.
Once you cycle enough, it no longer matters...

Most of the time, you are always cycling into the wind, uphill, both ways....
Title: Re: Problems with the Heliocentric Model
Post by: newhorizons on August 01, 2019, 04:11:35 PM
Quote
You displayed a lack of knowledge about the background which is why I did have to give you a lesson on Tycho and Kepler and the geocentric model . You showed this - post 24 on this thread .

OK you got me on that one... please advise where I showed this lack of knowledge as you put it.

I am still waiting on you to provide compelling evidence for what you believe to be a stationary Earth. I have already given examples for both the rotation of the Earth so please refer back through the discussion for that. You can go into whatever level of detail you wish to in terms of the astronomy side of things.  I will always be able to keep up with you. On that subject.. and just out of interest what experience do you have in using telescopes? 

Quote
there is no experimental/observational derivation of his supposed laws

Really... how do you come to that conclusion?  I can provide details of how you can reach an analytical derivation of Keplers 3rd law for example. I will gladly give you instructions if you would like to try it yourself.  I take it you do know what his 3rd law is about?
Title: Re: Problems with the Heliocentric Model
Post by: totallackey on August 01, 2019, 04:22:11 PM
Quote
You displayed a lack of knowledge about the background which is why I did have to give you a lesson on Tycho and Kepler and the geocentric model . You showed this - post 24 on this thread .

OK you got me on that one... please advise where I showed this lack of knowledge as you put it.

I am still waiting on you to provide compelling evidence for what you believe to be a stationary Earth. I have already given examples for both the rotation of the Earth so please refer back through the discussion for that. You can go into whatever level of detail you wish to in terms of the astronomy side of things.  I will always be able to keep up with you. On that subject.. and just out of interest what experience do you have in using telescopes? 

Quote
there is no experimental/observational derivation of his supposed laws

Really... how do you come to that conclusion?  I can provide details of how you can reach an analytical derivation of Keplers 3rd law for example. I will gladly give you instructions if you would like to try it yourself.  I take it you do know what his 3rd law is about?
I would prefer you take the actual math of Kepler, Einstein, and Newton, apply all of it into a CGI image of the supposed solar system and let's just see if it actually puts out on screen what is claimed by science.

I take it you know the math and all required inputs?
Title: Re: Problems with the Heliocentric Model
Post by: newhorizons on August 01, 2019, 06:24:05 PM
Quote
I would prefer you take the actual math of Kepler, Einstein, and Newton, apply all of it into a CGI image of the supposed solar system and let's just see if it actually puts out on screen what is claimed by science

I don't need to when it has already been done.  Check this out..

https://store.simulationcurriculum.com/products/starry-night-pro-plus-8

You have to buy this (which I have) and I can confirm that it is an excellent product. It does more than just paints pretty pictures of the night sky. It mathematically generates a simulation (=CGI image) of the night sky.  If you check the positions for Jupiters moons for example (or the visibility of the red spot for that matter) using the software and then compare these with what you actually see in a telescope you will find that it provides an almost exact match. So you can use it as an observation planner.

Jupiter is well positioned at the moment in the evening sky so if you have a pair of binoculars handy then I recommend you try it.  You can't see Jupiters GRS in binoculars but you can certainly see the four brightest satellites. Confirming computer predictions using direct observation.  I would say that qualifies as evidence wouldn't you?

I have yet to find any mathematically generated CGI images of a flat Earth however. If one exists then please point me to it.



Title: Re: Problems with the Heliocentric Model
Post by: newhorizons on August 01, 2019, 06:34:21 PM
Attached is a screenshot taken from my copy of SN8 to show the positions of the satellites and GRS of Jupiter for tonight.  You can see the GRS just to the left of Jupiters central meridian with the brightest satellites.  If it was clear enough tonight I would be glad to post an actual image of Jupiter taken through the telescope to compare it.
Title: Re: Problems with the Heliocentric Model
Post by: Tom Bishop on August 01, 2019, 06:49:39 PM
What makes you think that is based on the laws of Newton, rather than a pattern-based model?

See: https://wiki.tfes.org/Astronomical_Prediction_Based_on_Patterns

And : https://wiki.tfes.org/Three_Body_Problem
Title: Re: Problems with the Heliocentric Model
Post by: newhorizons on August 01, 2019, 07:15:57 PM
What makes you think it isn't Tom?

What are patterns based on? And where is the mathematically based CGI image of a flat Earth that seems to be very elusive.

Title: Re: Problems with the Heliocentric Model
Post by: Tom Bishop on August 01, 2019, 07:36:35 PM
What makes you think it isn't Tom?

What are patterns based on? And where is the mathematically based CGI image of a flat Earth that seems to be very elusive.

I think that astronomers and mathematicians have a hard time simulating systems with more than two bodies. There are plenty of people who think it is possible, or who link us to various visualizations and programs which might not be based on a full gravity simulation,  but I can't seem to find any statements in the official story telling us who solved it. The stories all seem to say that it's a difficult problem that was never solved.
Title: Re: Problems with the Heliocentric Model
Post by: newhorizons on August 01, 2019, 07:52:00 PM
So where does FET do it better where astronomers have failed then Tom? 

To be quite honest I don't really care how any of these simulations work.  Providing they can accurately predict consistently what I am likely to see in my telescope, that is all I need from it.  The mechanics behind it is not that important to me.

I referred earlier to Keplers laws. Specifically to the 3rd law which relates the period of a planets orbit with the distance of the planet from the Sun. This applies too all 8 major planets. You could interpret this as another example of one of your patterns I guess. There are patterns throughout nature and mathematics provides an excellent way of analysing and predicting such patterns.

The solar system comprises n bodies where n is a large and not precisely known number. It is a complex thing for mathematicians and astronomers to work with. But one way or another predicting astronomical events and phenomenon is something we seem to be quite good at now.  What are the planetary orbits if they are not patterns of one form or another regardless of how those patterns come about?
Title: Re: Problems with the Heliocentric Model
Post by: ChrisTP on August 01, 2019, 08:58:54 PM
What makes you think it isn't Tom?

What are patterns based on? And where is the mathematically based CGI image of a flat Earth that seems to be very elusive.

I think that astronomers and mathematicians have a hard time simulating systems with more than two bodies. There are plenty of people who think it is possible, or who link us to various visualizations and programs which might not be based on a full gravity simulation,  but I can't seem to find any statements in the official story telling us who solved it. The stories all seem to say that it's a difficult problem that was never solved.
It's interesting you mention this, if you recall I linked you to a simulation of 3 bodies (sun, earth and moon) where you could manipulate in real time the velocities, trajectories and mass of the bodies while they orbit;

https://phet.colorado.edu/sims/html/gravity-and-orbits/latest/gravity-and-orbits_en.html

All of this is based on maths and physics (and not predetermined keyframed animations) but I don't have to tell you that you might be sceptical about what's really going on in the code. As it happens, all the code is on github for you to check for yourself what's happening;

https://github.com/phetsims/gravity-and-orbits

If you don't fully understand (which is fine, no judgement) I'd recommend getting someone you trust who does understand to explain it. I haven't fully gone over all of it myself and it's does have a fair few third party dependencies which should also be listed and on github. You could also find the people listed in the credits and maybe ask them about it for more info but to anyone reading this please don't harass them (disclaimer).

Feel free to see if there is actually any foul play going on and if you do find anything that would be 'faking' it do tell, I'd be interested to find out too.
Title: Re: Problems with the Heliocentric Model
Post by: newhorizons on August 01, 2019, 09:07:13 PM
Those on the FET side regularly use the 3 body problem as their favourite line of attack for questioning the validity of the heliocentric model. But I have yet to see anything better come from their side to stand up as a viable alternative.

Somerled seems to be convinced that the lack of any direct sensation of Earth motion provides the best evidence for the earth being stationary. Now consider an astronaut located somewhere in interplanetary or even interstellar space travelling at say 17,500 miles an hour.  Looking around him, how would the astronaut be able to detect whether or not he was moving at all? He wouldn't be able to feel himself moving.  Furthermore since there will be nothing but stars all around him in which ever direction he looks, how can he have any sense of direction as we would define it?

We now place the astronaut on a spherical rocky body. How does the astronaut tell whether or not the body is moving in space or not? He can walk about on the body randomly forever without ever reaching an edge or a boundary to the surface. He will see different patterns of stars depending on where on the surface he is standing. The only place he cannot see any stars is directly 'below' him because the body is hiding them from view.
Title: Re: Problems with the Heliocentric Model
Post by: Tom Bishop on August 01, 2019, 09:39:20 PM
What makes you think it isn't Tom?

What are patterns based on? And where is the mathematically based CGI image of a flat Earth that seems to be very elusive.

I think that astronomers and mathematicians have a hard time simulating systems with more than two bodies. There are plenty of people who think it is possible, or who link us to various visualizations and programs which might not be based on a full gravity simulation,  but I can't seem to find any statements in the official story telling us who solved it. The stories all seem to say that it's a difficult problem that was never solved.
It's interesting you mention this, if you recall I linked you to a simulation of 3 bodies (sun, earth and moon) where you could manipulate in real time the velocities, trajectories and mass of the bodies while they orbit;

https://phet.colorado.edu/sims/html/gravity-and-orbits/latest/gravity-and-orbits_en.html

All of this is based on maths and physics (and not predetermined keyframed animations) but I don't have to tell you that you might be sceptical about what's really going on in the code. As it happens, all the code is on github for you to check for yourself what's happening;

https://github.com/phetsims/gravity-and-orbits

If you don't fully understand (which is fine, no judgement) I'd recommend getting someone you trust who does understand to explain it. I haven't fully gone over all of it myself and it's does have a fair few third party dependencies which should also be listed and on github. You could also find the people listed in the credits and maybe ask them about it for more info but to anyone reading this please don't harass them (disclaimer).

Feel free to see if there is actually any foul play going on and if you do find anything that would be 'faking' it do tell, I'd be interested to find out too.

You linked me to a small Earth Science applet for middle school students: https://phet.colorado.edu/en/contributions/view/3566
Title: Re: Problems with the Heliocentric Model
Post by: ChrisTP on August 01, 2019, 09:56:39 PM
What makes you think it isn't Tom?

What are patterns based on? And where is the mathematically based CGI image of a flat Earth that seems to be very elusive.

I think that astronomers and mathematicians have a hard time simulating systems with more than two bodies. There are plenty of people who think it is possible, or who link us to various visualizations and programs which might not be based on a full gravity simulation,  but I can't seem to find any statements in the official story telling us who solved it. The stories all seem to say that it's a difficult problem that was never solved.
It's interesting you mention this, if you recall I linked you to a simulation of 3 bodies (sun, earth and moon) where you could manipulate in real time the velocities, trajectories and mass of the bodies while they orbit;

https://phet.colorado.edu/sims/html/gravity-and-orbits/latest/gravity-and-orbits_en.html

All of this is based on maths and physics (and not predetermined keyframed animations) but I don't have to tell you that you might be sceptical about what's really going on in the code. As it happens, all the code is on github for you to check for yourself what's happening;

https://github.com/phetsims/gravity-and-orbits

If you don't fully understand (which is fine, no judgement) I'd recommend getting someone you trust who does understand to explain it. I haven't fully gone over all of it myself and it's does have a fair few third party dependencies which should also be listed and on github. You could also find the people listed in the credits and maybe ask them about it for more info but to anyone reading this please don't harass them (disclaimer).

Feel free to see if there is actually any foul play going on and if you do find anything that would be 'faking' it do tell, I'd be interested to find out too.

You linked me to a small Earth Science applet for middle school students: https://phet.colorado.edu/en/contributions/view/3566
Sorry, I'm not sure I understand what your point is here? Are you saying because it's for middle school it's untrustworthy? Or that it's somehow invalidated because it's lower form education? Did you happen to look over the code for this mathematical representation of orbiting bodies already and find foul play?

What difference does it make if the code was made to represent orbits for middle schoolers? Is it or is it not a 3 body orbit based on maths playing out in real time (not predetermined)? You're gunna have to give me more info here I think.
Title: Re: Problems with the Heliocentric Model
Post by: Tom Bishop on August 01, 2019, 10:25:00 PM
I don't see that it claims to be a three body problem simulator. It's just an application for a middle school curriculum, like their sophisticated carpet simulator "John Travoltage". (https://phet.colorado.edu/sims/html/john-travoltage/latest/john-travoltage_en.html)

(https://s3.gifyu.com/images/ezgif.com-optimize-256191727ec4f0514.gif)
Title: Re: Problems with the Heliocentric Model
Post by: ChrisTP on August 01, 2019, 11:23:04 PM
I don't see that it claims to be a three body problem simulator. It's just an application for a middle school curriculum, like their sophisticated carpet simulator "John Travoltage". (https://phet.colorado.edu/sims/html/john-travoltage/latest/john-travoltage_en.html)

(https://s3.gifyu.com/images/ezgif.com-optimize-256191727ec4f0514.gif)
This is like saying if you're a terrible carpenter then you're a terrible Knitter... It's cool that you found other representations on their website and all, but maybe focus on the one I linked to you, with all of it's code included.
Title: Re: Problems with the Heliocentric Model
Post by: totallackey on August 02, 2019, 10:19:32 AM
Quote
I would prefer you take the actual math of Kepler, Einstein, and Newton, apply all of it into a CGI image of the supposed solar system and let's just see if it actually puts out on screen what is claimed by science

I don't need to when it has already been done.  Check this out..

https://store.simulationcurriculum.com/products/starry-night-pro-plus-8

You have to buy this (which I have) and I can confirm that it is an excellent product. It does more than just paints pretty pictures of the night sky. It mathematically generates a simulation (=CGI image) of the night sky.  If you check the positions for Jupiters moons for example (or the visibility of the red spot for that matter) using the software and then compare these with what you actually see in a telescope you will find that it provides an almost exact match. So you can use it as an observation planner.

Jupiter is well positioned at the moment in the evening sky so if you have a pair of binoculars handy then I recommend you try it.  You can't see Jupiters GRS in binoculars but you can certainly see the four brightest satellites. Confirming computer predictions using direct observation.  I would say that qualifies as evidence wouldn't you?

I have yet to find any mathematically generated CGI images of a flat Earth however. If one exists then please point me to it.
I would still appreciate it if you could confirm the the product you offer utilizes the math utilized by Kepler, Newton, and Einstein.

To do that, you would need to publish the inputs.
Title: Re: Problems with the Heliocentric Model
Post by: totallackey on August 02, 2019, 10:21:13 AM
What makes you think it isn't Tom?

What are patterns based on? And where is the mathematically based CGI image of a flat Earth that seems to be very elusive.
Just a guess...

What makes him think it is simply pattern inputs is the fact if you publish the code, you won't find anything related to Kepler, Newton, or Einstein.
Title: Re: Problems with the Heliocentric Model
Post by: totallackey on August 02, 2019, 10:48:36 AM
Sorry, I'm not sure I understand what your point is here? Are you saying because it's for middle school it's untrustworthy? Or that it's somehow invalidated because it's lower form education? Did you happen to look over the code for this mathematical representation of orbiting bodies already and find foul play?

What difference does it make if the code was made to represent orbits for middle schoolers? Is it or is it not a 3 body orbit based on maths playing out in real time (not predetermined)? You're gunna have to give me more info here I think.
It would be helpful for me, anyway, for you to definitively establish the code utilized for this model is, in fact, based on the math presented by Kepler, Newton, and Einstein.
Title: Re: Problems with the Heliocentric Model
Post by: newhorizons on August 02, 2019, 11:50:11 AM
@ Totallackey

I cant confirm what you say because it is not my product. If you want to know more information about the mechanics going on under the surface then I can only recommend you take that up with the developer. All I know is that I have been using SN since version 4 and I know it works.

Incidentally I also use another data source. The Horizons web interface from JPL which liknks into the software that controls my mount. That allows me to track objects like comets, asteroids and even satellites very accurately. They do not follow the same paths as the stars, Moon and planets. So Horizons feeds position data on a per minute basis to the mount so it can track anywhere in the sky as quickly as it needs to to keep a target centered.
Title: Re: Problems with the Heliocentric Model
Post by: Tim Alphabeaver on August 02, 2019, 02:38:16 PM
What makes you think that is based on the laws of Newton, rather than a pattern-based model?
I asked you in a different thread about astronomical observations that cannot be pattern-based, such as calculating the trajectory of an asteroid that's coming close to Earth. How could you calculate a close-approach of an asteroid if it's the first time this object has encountered Earth?

What kind of pattern are you talking about here, exactly? The Newtonian approach would of course be simple if you have enough data about the asteroid's velocity and position - you can just whack it into your favourite equation solver and calculate its position at future times to arbitrarily high accuracy with a large enough computer.
Title: Re: Problems with the Heliocentric Model
Post by: AATW on August 02, 2019, 02:49:45 PM
What makes you think it isn't Tom?

What are patterns based on? And where is the mathematically based CGI image of a flat Earth that seems to be very elusive.

I think that astronomers and mathematicians have a hard time simulating systems with more than two bodies. There are plenty of people who think it is possible, or who link us to various visualizations and programs which might not be based on a full gravity simulation,  but I can't seem to find any statements in the official story telling us who solved it. The stories all seem to say that it's a difficult problem that was never solved.
What you constantly fail to understand on this topic is that while the 'n body problem' has no analytical solution, it can be broken down into multiple 2 body problems which do have an analytical solution and then use numerical methods to solve it. That does make it an imperfect model but a model doesn't have to be perfect to be useful. Einstein showed that the Newtonian model is imperfect, it breaks down at speeds close to the speed of light, when masses get big and so on. For most practical purposes though it's adequate.
Title: Re: Problems with the Heliocentric Model
Post by: Tom Bishop on August 02, 2019, 03:16:43 PM
What makes you think it isn't Tom?

What are patterns based on? And where is the mathematically based CGI image of a flat Earth that seems to be very elusive.

I think that astronomers and mathematicians have a hard time simulating systems with more than two bodies. There are plenty of people who think it is possible, or who link us to various visualizations and programs which might not be based on a full gravity simulation,  but I can't seem to find any statements in the official story telling us who solved it. The stories all seem to say that it's a difficult problem that was never solved.
What you constantly fail to understand on this topic is that while the 'n body problem' has no analytical solution, it can be broken down into multiple 2 body problems which do have an analytical solution and then use numerical methods to solve it. That does make it an imperfect model but a model doesn't have to be perfect to be useful. Einstein showed that the Newtonian model is imperfect, it breaks down at speeds close to the speed of light, when masses get big and so on. For most practical purposes though it's adequate.

Find a source other than your own opinion. Show us where it says that the Sun-Earth-Moon system is able to be simulated with the Three Body Problem.

All of the mainstream sources say that it's impossible: https://wiki.tfes.org/Three_Body_Problem

The "numerical solutions" to the three body problem that you read references to are the figure eight and other highly symmetric solutions.
Title: Re: Problems with the Heliocentric Model
Post by: AATW on August 02, 2019, 03:32:33 PM
Find a source other than your own opinion. Show us where it says that the Sun-Earth-Moon system is able to be simulated with the Three Body Problem.

OK:

https://en.wikipedia.org/wiki/Numerical_model_of_the_Solar_System

Quote
All of the mainstream sources say that it's impossible: https://wiki.tfes.org/Three_Body_Problem

An analytical solution is beyond us. A numerical solution which gives results good enough to be useful is possible with modern computing power.
Title: Re: Problems with the Heliocentric Model
Post by: Tom Bishop on August 02, 2019, 05:25:58 PM
'Numerical' is a generic term and does not refer to the Three Body Problem. Ptolmy uses 'numerical computation' in the Almagest. Was he solving Newton's Laws and Three Body Problems?

https://books.google.com/books?id=JVhTtVA2zr8C&pg=PA29&source=gbs_toc_r&cad=4#v=onepage&q&f=false

(https://i.imgur.com/tLawyDV.png)

---

Dr. Gopi Krishna Vijaya says that Newtonian astronomers use perturbations/epicycles with a gravitational disguise.

https://reciprocalsystem.org/PDFa/Replacing%20the%20Foundations%20of%20Astronomy%20(Vijaya,%20Gopi%20Krishna).pdf

Epicycles Once More

“ Following the Newtonian era, in the 18th century there were a series of mathematicians – Bernoulli, Clairaut, Euler, D’Alembert, Lagrange, Laplace, Leverrier – who basically picked up where Newton left off and ran with it. There were no descendants to the wholistic viewpoints of Tycho and Kepler, but only those who made several improvements of a mathematical nature to Newtonian theory. Calculus became a powerful tool in calculating the effects of gravitation of all the planets upon each other, due to their assumed masses. The motion of the nearest neighbor – the Moon – was a surprisingly hard nut to crack even for Newton, and several new mathematical techniques had to be invented just to tackle that.

In the process, a new form of theory became popular: Perturbation theory. In this approach, a small approximate deviation from Newton's law is assumed, based on empirical data, and then a rigorous calculation of differential equation is used to nail down the actual value of the deviation. It does not take much to recognize that this was simply the approach taken before Kepler by Copernicus and others for over a thousand years – adding epicycles to make the observations fit. It is the same concept, but now dressed up in gravitational disguise: ”

(https://i.imgur.com/KiTaMfy.png)

“ In other words, the entire thought process took several steps backwards, to redo the same process as the Ptolemaic - Copernican epicycle theory, only with different variables. The more logical way of approach would have been to redirect the focus of the improved mathematical techniques to the assumptions in Newton’s theory, but instead the same equations were re-derived with calculus, without examining the assumptions. Hence any modern day textbook gives the same derivation for circular and elliptical motion that Newton first derived in his Principia. The equivalence of the epicycle theory and gravitational theory has not been realized, and any new discovery that fits in with the mathematical framework of Newtonian gravity is lauded as a “triumph of the theory of gravitation.” In reality, it is simply the triumph of fitting curves to the data or minor linear extrapolations – something that had already been done at least since 2nd century AD. Yet the situation is conceptually identical. ”

From the VSOP talk page, the solar system model which is linked at the bottom of the page you provided:

“ Modelling VSOP on a ubiquitous PC computer program, starting with only one element for each of the three parameters (L, B R) and then slowly incrementing the number of elements, gives a sense of irony that it is in fact nothing more than a more complex development of the ancient deferent / epicycle system used by Ptolemy. A system that despite being totally dismissed out of hand for being intellectually "wrong", was able to provide a prediction service accurate enough to match the observational resolution available (naked eye, with no reliable mechanical timekeeping). A system that, astoundingly to this author, was able to detect and measure, accurately, the lunar evection, one of the still-used perturbations of the Earth-Moon system. Summing powers of sines and cosines is certainly tantamount to circles upon (or perhaps within) circles; recursing, or perhaps simply nesting, almost endlessly. Whilst of course this is totally irrelevant to the mathematics, it perhaps behoves Wikipedia's wider terms of reference to include this as a philosophical point. ”

All of this tells me that it's not truly a simulation of gravity.

Dr. Vijaya says:

" The Dead End

In the late 19th century, one of the French mathematicians – Henri Poincaré – had already discovered that many of the terms being used in the “perturbation” series by mathematicians like Laplace and Lagrange were becoming infinite for long periods of time, making the system unstable. In simple words, the solutions ‘blow up’ fairly quickly. He also showed that the general problem of 3 mutually gravitating bodies was insoluble through any mathematical analysis! Many physicists and mathematicians built up modern “Chaos theory” based on these ideas, to show simply that one cannot calculate the movements of the planets accurately. Thus began the field of non-linear
dynamics.

In the middle of the 20th century, with computers entering the field, the mathematicians pretty much gave up on calculating the orbits by themselves and programmed the computer to do it, even though it was mathematically shown that these orbits were incalculable. They had to be satisfied with approximations or numerical methods (or “brute force” methods.) The result of it all was that after 300 years, Newtonian/Einsteinian thought lands in the same spot that Kepler ended: the orbits point to a living or chaotic system. Only now, there is the additional baggage of all the wrong concepts introduced with regard to “inverse-square law”, “gravitational attraction”, “gravitational mass” and “curved space-time” along with uncountable number of minor assumptions. In this process, an enormous amount of human effort was put to derive thousands of terms in equations over centuries. The entire enterprise has been a wild goose chase "
Title: Re: Problems with the Heliocentric Model
Post by: newhorizons on August 02, 2019, 07:11:07 PM
I get it that you are clearly dedicated to the cause of proving that the likes of Newton, Kepler and Einstein didn't know what they were talking about.  Fair enough, I will take your word for it.

However, how do you account for the fact that modern software simulations such as that I have mentioned are quite capable of correctly predicting what we see in the sky? I'm guessing that you would dismiss modelling based on the 3 body problem so what methods to you think the software uses to achieve what it does very well?  Computers only speak the language of mathematics and binary maths at that. So any patterns that are used would have to be mathematically generated. I know it works because I use it all the time and it exactly matches what I see in the sky. So perhaps you guys can tell me how it does this?
Title: Re: Problems with the Heliocentric Model
Post by: AATW on August 03, 2019, 08:55:05 PM
'Numerical' is a generic term and does not refer to the Three Body Problem.
It doesn't refer to any particular problem. It's just a method of solving certain problems where there is no perfect analytical solution.
The three body problem is an example where it can be used.
Equations which perfectly predict the future state of the system are beyond us. So there's no equations you can plug in the initial positions and velocities and then plug in a parameter for time, 't' and the solution to those equations is the future velocities and positions.
But numerical techniques which break the problem down and solve it for a number of time steps do provide, with modern computing power, very good results
Another link here:

https://www.quora.com/How-can-the-solar-systems-behavior-be-correctly-simulated

That link references this, a very technical paper on the subject:

https://arxiv.org/pdf/1208.0689.pdf

Quote
Dr. Gopi Krishna Vijaya says that Newtonian astronomers use perturbations/epicycles with a gravitational disguise.
Yeah. In the 18th century. So?
We are talking about how simulations work now. They didn't have the raw computing power which we do now and which enables numerical solutions to be effective and accurate.

Later in the paper you linked to it says:

Quote
In the middle of the 20th century, with computers entering the field, the mathematicians pretty much gave up on
calculating the orbits by themselves and programmed the computer to do it, even though it was mathematically
shown that these orbits were incalculable. They had to be satisfied with approximations or numerical methods (or
“brute force” methods.)

I see you quoted that bit too which is odd as it backs up exactly what I'm claiming.
Sorry Tom but this is another example of you just not understanding something as well as you think you do.
Title: Re: Problems with the Heliocentric Model
Post by: Tom Bishop on August 03, 2019, 10:00:08 PM
What you posted is using perturbation theory. Do a search for "perturb" in the top quora answer link and the pdf you posted.

You claim that perturbations were only used in the 18th century and then link us to a PDF with a bunch of references to the use of perturbations. Odd logic. Almost as if you just blindly posted the first thing you googled.

It's not enough to find something that says numerical. The perturbation and epicycles methods are also numerical.
Title: Re: Problems with the Heliocentric Model
Post by: AATW on August 04, 2019, 08:54:55 AM
What you posted is using perturbation theory.

And what is that?

Quote
Perturbation theory comprises mathematical methods for finding an approximate solution to a problem, by starting from the exact solution of a related, simpler problem. A critical feature of the technique is a middle step that breaks the problem into "solvable" and "perturbation" parts. Perturbation theory is applicable if the problem at hand cannot be solved exactly, but can be formulated by adding a "small" term to the mathematical description of the exactly solvable problem.

Its relevance to celestial simulations:

Quote
Since the planets are very remote from each other, and since their mass is small as compared to the mass of the Sun, the gravitational forces between the planets can be neglected, and the planetary motion is considered, to a first approximation, as taking place along Kepler's orbits, which are defined by the equations of the two-body problem, the two bodies being the planet and the Sun.
Since astronomic data came to be known with much greater accuracy, it became necessary to consider how the motion of a planet around the Sun is affected by other planets. This was the origin of the three-body problem; thus, in studying the system Moon–Earth–Sun the mass ratio between the Moon and the Earth was chosen as the small parameter. Lagrange and Laplace were the first to advance the view that the constants which describe the motion of a planet around the Sun are "perturbed", as it were, by the motion of other planets and vary as a function of time; hence the name "perturbation theory".
Perturbation theory was investigated by the classical scholars—Laplace, Poisson, Gauss—as a result of which the computations could be performed with a very high accuracy. The discovery of the planet Neptune in 1848 by Urbain Le Verrier, based on the deviations in motion of the planet Uranus (he sent the coordinates to Johann Gottfried Galle who successfully observed Neptune through his telescope), represented a triumph of perturbation theory.

https://en.wikipedia.org/wiki/Perturbation_theory

So yeah, it is a numerical method, it's not just about cycles
And the discovery of Neptune came about because if it - a powerful vindication of the method and the heliocentric model it's modelling.
Title: Re: Problems with the Heliocentric Model
Post by: totallackey on August 05, 2019, 10:35:36 AM
I get it that you are clearly dedicated to the cause of proving that the likes of Newton, Kepler and Einstein didn't know what they were talking about.  Fair enough, I will take your word for it.

However, how do you account for the fact that modern software simulations such as that I have mentioned are quite capable of correctly predicting what we see in the sky? I'm guessing that you would dismiss modelling based on the 3 body problem so what methods to you think the software uses to achieve what it does very well?  Computers only speak the language of mathematics and binary maths at that. So any patterns that are used would have to be mathematically generated. I know it works because I use it all the time and it exactly matches what I see in the sky. So perhaps you guys can tell me how it does this?
Again, I think you misunderstand.

I do not think Tom is, "...clearly dedicated to the cause of proving that the likes of Newton, Kepler and Einstein didn't know what they were talking about," and I know I am certainly not.

The point is, these three are trotted out like Lippizaners and we are told, "...here's the bona fide math."

Well, math is math certainly.

Computers are the IDEAL tool for RENDERING VISUAL OUTPUTS of math.

Yet, when challenged for such a simple demonstration of these wonderful maths, crickets...
Title: Re: Problems with the Heliocentric Model
Post by: newhorizons on August 05, 2019, 04:21:12 PM
Crickets...?

My point is I don't need to know the mechanics and physics of an internal combustion engine to drive my car. As long as it works I'm happy.  Similarly the mechanics and math behind how celestial simulators work would be interesting to know, but I don't need to know it to use the software do I.  As long as I can fire up the software, connect it to my telescope mount (using ASCOM) and get it to point accurately to my targets that is enough.  It satisfies a function.
Title: Re: Problems with the Heliocentric Model
Post by: totallackey on August 06, 2019, 10:44:01 AM
Crickets...?

My point is I don't need to know the mechanics and physics of an internal combustion engine to drive my car. As long as it works I'm happy.  Similarly the mechanics and math behind how celestial simulators work would be interesting to know, but I don't need to know it to use the software do I.  As long as I can fire up the software, connect it to my telescope mount (using ASCOM) and get it to point accurately to my targets that is enough.  It satisfies a function.
You are not driving the solar system, though, and the operation of your car does depend, in some way, on people who do know the mechanics and physics of automobiles in general, so spare everyone the strawman comparison.

You made claims:
1) Einstein, Kepler, and Newton all have math explaining how it is possible for all of the solar system to exist as depicted in science textbooks;
B) These maths work in concert;
III) There are computer simulations showing the solar system AS IS to good people of the earth and these simulations utilize the math of Kepler, Newton, and Einstein for such simulation and rendering.

Me?

I stated computers were an excellent tool for rendering MATH in VISUAL FORM (a statement of indisputable veracity), and asked you to present evidence of just such evidence...a rendered simulation of the solar system, based on the combined math of Kepler, Newton, and Einstein...

Crickets...

It would nice for once, if you simply and readily admit these simulations are not utilizing math from Kepler, Newton, and Einstein, and EVEN IF THEY WERE, you wouldn't have a clue.

Thank you.
Title: Re: Problems with the Heliocentric Model
Post by: newhorizons on August 06, 2019, 02:09:31 PM
As I said previously you would have to ask the developers of the software the framework around which it is designed. They might or might not be taking into account the maths of Newton etc. It really doesn't matter to me.

What I do know is that the positions of natural satellites and artificial satellites is presented very accurately. Furthermore data on new objects such as comets etc is added each time you load the software. So my guess is that several data sources are used. I believe you can download a free copy for trial purposes if you'd like to test it out.
Title: Re: Problems with the Heliocentric Model
Post by: totallackey on August 06, 2019, 03:35:37 PM
As I said previously you would have to ask the developers of the software the framework around which it is designed. They might or might not be taking into account the maths of Newton etc. It really doesn't matter to me.

What I do know is that the positions of natural satellites and artificial satellites is presented very accurately. Furthermore data on new objects such as comets etc is added each time you load the software. So my guess is that several data sources are used. I believe you can download a free copy for trial purposes if you'd like to test it out.
So, if in testing it is clearly demonstrated the inputs of these programs include nothing containing the math from Kepler, Newton, and Einstein, where does that leave you?
Title: Re: Problems with the Heliocentric Model
Post by: newhorizons on August 06, 2019, 04:46:34 PM
It leaves me exactly where I am now. As I said feel free to contact Simulation Curriculum yourself and see if they will give you details of the inner workings of Starry Night and the other products they offer then feel free. If you have an iPad you might like to give SkySafari a try. It is much cheaper but equally accurate in my experience.

An aspect of FET which I can't quite fathom is the assertion that the stars are much closer than they actually are. There are several ways in which that can be demonstrated beyond reasonable doubt. Tom has also mentioned how some stars apparently have diameters about one tenth of the Moons diameter. According to some FE Theorists anyway. Which ones are they?
Title: Re: Problems with the Heliocentric Model
Post by: ChrisTP on August 06, 2019, 05:31:13 PM
I think if there is a 3d software that shows the solar system which matches our own observations then the ball is really in the flat earthers court, either make a 3D representation of the flat earth that matches observations or stop asking whether or not software like Starry Night uses the correct math. What does that matter when it matches what we see and is a globe earth representation? When there is a 3D representation of the flat earth that matches the skies and land exactly as we observe in real life then you can question what math was used to make a globe version. Until then it's kind of a moot point
Title: Re: Problems with the Heliocentric Model
Post by: newhorizons on August 06, 2019, 06:42:20 PM
Exactly. I've certainly got nothing to prove. I've been using SN for years and it has always worked for the purpose that I use it for. When I first tried it out I didn't immediately start asking myself 'Wow.. I wonder if this is based on celestial and orbital mechanics as predicted by Newton or Kepler...if not I stop using it right now!'

I have never seen a version of SN that is based on FET but I'm sure if those in the know on such things were to contact a software developer and present a strong enough case that convinced them it was worth the investment in terms of time and budget then I'm sure they would do it. Whether they could make it work or not is another matter!  I'm sure it was Tom who admitted sometime ago that not much is known about the celestial objects. Someone on here has it attached to their signature. In that case isn't it about time you caught up!
Title: Re: Problems with the Heliocentric Model
Post by: totallackey on August 07, 2019, 10:24:55 AM
I think if there is a 3d software that shows the solar system which matches our own observations then the ball is really in the flat earthers court, either make a 3D representation of the flat earth that matches observations or stop asking whether or not software like Starry Night uses the correct math. What does that matter when it matches what we see and is a globe earth representation? When there is a 3D representation of the flat earth that matches the skies and land exactly as we observe in real life then you can question what math was used to make a globe version. Until then it's kind of a moot point
There is such a 3D representation of the flat earth skies.

It is called a planetarium.

The floor is flat and the roof is a dome.

Matches perfectly what we see in the skies above and also doesn't use Kepler, Newton, or Einstein, in the modeling of the skies above.

So again, the ball is in the RE court.

According to RE adherents, the entire skies above are operating under the math and laws of Kepler, Newton, and Einstein.

When asked to produce a mathematical visual rendering of these movements, RE has not (because they cannot).
Title: Re: Problems with the Heliocentric Model
Post by: newhorizons on August 07, 2019, 11:43:21 AM
If you think planetariums are based on the assertion that the Earth is flat then you are mistaken. Just turn on the option to view the celestial grid.

Of course the representation of the ground in a planetarium is a flat surface. But that doesnt mean the entire Earth is flat does it.

The ball remains very firmly in the court of the flat Earthers. By the way did you see my own images of the Moon I posted under Moon Appearance? I was tracking the Moon using Starry Night at the time. I also used the software to aim at the Moon. Spot on as always.
Title: Re: Problems with the Heliocentric Model
Post by: totallackey on August 07, 2019, 11:50:29 AM
If you think planetariums are based on the assertion that the Earth is flat then you are mistaken. Just turn on the option to view the celestial grid.
A celestial grid does not reflect upon the shape of the surface.

So, yeah, the observing surface being flat is based on the same viewing surface you have outdoors.
Of course the representation of the ground in a planetarium is a flat surface. But that doesnt mean the entire Earth is flat does it.
I contend it does.
The ball remains very firmly in the court of the flat Earthers. By the way did you see my own images of the Moon I posted under Moon Appearance? I was tracking the Moon using Starry Night at the time. I also used the software to aim at the Moon. Spot on as always.
Yeah, but nothing in Starry Night utilizes Kepler, Newton, or Einstein, math or laws, as you admit.

All of these programs simply enter some form of math to render visual outputs.

That math, since it has nothing to do with Kepler, Newton, or Einstein, is by default NOT based on RE dogma or the heliocentric model.
Title: Re: Problems with the Heliocentric Model
Post by: newhorizons on August 07, 2019, 12:16:02 PM
You can contend whatever you wish.

The celestial grid you might have noticed is like looking at the inside of a huge and imaginary sphere. This is an illusion based on the fact that you cannot determine the various distances of celestial objects directly.

I haven't admitted anything of the sort. If I knew either way I would have told you but since I didn't develop the software how can I possibly know? I suggested you contact the company who does produce SN so you can find out for yourself. Have you done that yet?

SN is predicting a shadow transit of Jupiters moon Io tonight. I will take an image of the real Jupiter tonight with my telescope and see how close the two match. I will post the image for you to see. Providing skies are clear of course.
Title: Re: Problems with the Heliocentric Model
Post by: totallackey on August 07, 2019, 12:27:34 PM
You can contend whatever you wish.

The celestial grid you might have noticed is like looking at the inside of a huge and imaginary sphere. This is an illusion based on the fact that you cannot determine the various distances of celestial objects directly.
But it's not imaginary.

That is why a planetarium has a dome.

Your program at home has no dome, but it can render one.
I haven't admitted anything of the sort. If I knew either way I would have told you but since I didn't develop the software how can I possibly know? I suggested you contact the company who does produce SN so you can find out for yourself. Have you done that yet?
Intellectual honesty places that burden on you.

You introduced the software as evidence of the heliocentric model.

If it is, then the routines governing its operation, by default, are based on the laws and maths proposed by Kepler, Newton, and Einstein.

The reason you won't contact the developers of the software is simple.

You know the emperor is wearing no clothes.

So yes, by failing to to research fully the evidence you offer,or even asking the developers regarding the math inputs, you have admitted something.

There is nothing wrong with admitting that, by the way.
SN is predicting a shadow transit of Jupiters moon Io tonight. I will take an image of the real Jupiter tonight with my telescope and see how close the two match. I will post the image for you to see. Providing skies are clear of course.
Great!

Again, that is based on past performance and not some future predictor offered by anyone.
Title: Re: Problems with the Heliocentric Model
Post by: robinofloxley on August 07, 2019, 01:21:03 PM
You introduced the software as evidence of the heliocentric model.

If it is, then the routines governing its operation, by default, are based on the laws and maths proposed by Kepler, Newton, and Einstein.

The reason you won't contact the developers of the software is simple.

You know the emperor is wearing no clothes.

If I understand you correctly you don't believe that planetarium software makes use of Kepler's equations. Well Starry Night is commercial software, so you can't easily look at the source code, however there is a very good open source alternative Stellarium and you can easily check the code for yourself.

https://github.com/Stellarium/stellarium/blob/master/plugins/Satellites/src/gsatellite/sgp4ext.cpp (https://github.com/Stellarium/stellarium/blob/master/plugins/Satellites/src/gsatellite/sgp4ext.cpp)

Take a look at line 159 onwards:

Quote
function newtonnu ... this function solves keplers equation...

Basically it's using Kepler's equation and a simplified perturbation model SGP4 https://en.wikipedia.org/wiki/Simplified_perturbations_models (https://en.wikipedia.org/wiki/Simplified_perturbations_models) for satellite calculations.

I can't speak for Starry Night, but certainly something in Stellarium utilizes Kepler.

If I were sitting in a candlelit room with a quill pen and some trig tables trying to work out a planetary position, I'd no doubt be trying to use Kepler's equation, however these days we have powerful computers and we have far more sophisticated models which use numerical methods to deal with the perturbations inherent in the N-body problem.

For obvious reasons the JPL have a vested interest in celestial mechanics and they have produced a whole series of more and more refined models over time https://en.wikipedia.org/wiki/Jet_Propulsion_Laboratory_Development_Ephemeris (https://en.wikipedia.org/wiki/Jet_Propulsion_Laboratory_Development_Ephemeris).

So if I were trying to write my own planetarium software today and wanted really accurate results, my starting point would be VSOP87 https://en.wikipedia.org/wiki/VSOP_(planets) (https://en.wikipedia.org/wiki/VSOP_(planets)) which is based on JPL's DE200 ephemeris model. This is what Stellarium and a number of other similar projects use, I don't know about Starry Night.

Using VSOP87 is reasonably straightforward. Understanding it less so. http://neoprogrammics.com/vsop87/vsop87_theory_paper/VSOP87%20Theory%20Paper.pdf (http://neoprogrammics.com/vsop87/vsop87_theory_paper/VSOP87%20Theory%20Paper.pdf) however it certainly does include a lot of math!

The section of this paper explaining how the VSOP87 solution is derived states:

Quote
We must first solve Kepler's equation in order to get the expressions of the variables X, Y, Z.

So it seems that VSOP87 (upon which Stellarium's planetary positioning is based) does indeed make use of a lot of math and Kepler's equation.
Title: Re: Problems with the Heliocentric Model
Post by: totallackey on August 07, 2019, 01:44:45 PM
You introduced the software as evidence of the heliocentric model.

If it is, then the routines governing its operation, by default, are based on the laws and maths proposed by Kepler, Newton, and Einstein.

The reason you won't contact the developers of the software is simple.

You know the emperor is wearing no clothes.

If I understand you correctly you don't believe that planetarium software makes use of Kepler's equations. Well Starry Night is commercial software, so you can't easily look at the source code, however there is a very good open source alternative Stellarium and you can easily check the code for yourself.

https://github.com/Stellarium/stellarium/blob/master/plugins/Satellites/src/gsatellite/sgp4ext.cpp (https://github.com/Stellarium/stellarium/blob/master/plugins/Satellites/src/gsatellite/sgp4ext.cpp)

Take a look at line 159 onwards:

Quote
function newtonnu ... this function solves keplers equation...

Basically it's using Kepler's equation and a simplified perturbation model SGP4 https://en.wikipedia.org/wiki/Simplified_perturbations_models (https://en.wikipedia.org/wiki/Simplified_perturbations_models) for satellite calculations.

I can't speak for Starry Night, but certainly something in Stellarium utilizes Kepler.
Simply referring to Kepler or Newton does not in fact mean the program utilizes Kepler or Newton.

It certainly makes no reference to whether this particular section of code includes a resultant modeling output of all of the solar system, or even three body modeling.

So yes, while it makes mention, there is no real evidence those maths or equations are even utilized.
If I were sitting in a candlelit room with a quill pen and some trig tables trying to work out a planetary position, I'd no doubt be trying to use Kepler's equation, however these days we have powerful computers and we have far more sophisticated models which use numerical methods to deal with the perturbations inherent in the N-body problem.

For obvious reasons the JPL have a vested interest in celestial mechanics and they have produced a whole series of more and more refined models over time https://en.wikipedia.org/wiki/Jet_Propulsion_Laboratory_Development_Ephemeris (https://en.wikipedia.org/wiki/Jet_Propulsion_Laboratory_Development_Ephemeris).

So if I were trying to write my own planetarium software today and wanted really accurate results, my starting point would be VSOP87 https://en.wikipedia.org/wiki/VSOP_(planets) (https://en.wikipedia.org/wiki/VSOP_(planets)) which is based on JPL's DE200 ephemeris model. This is what Stellarium and a number of other similar projects use, I don't know about Starry Night.

Using VSOP87 is reasonably straightforward. Understanding it less so. http://neoprogrammics.com/vsop87/vsop87_theory_paper/VSOP87%20Theory%20Paper.pdf (http://neoprogrammics.com/vsop87/vsop87_theory_paper/VSOP87%20Theory%20Paper.pdf) however it certainly does include a lot of math!

The section of this paper explaining how the VSOP87 solution is derived states:

Quote
We must first solve Kepler's equation in order to get the expressions of the variables X, Y, Z.

So it seems that VSOP87 (upon which Stellarium's planetary positioning is based) does indeed make use of a lot of math and Kepler's equation.
It makes use of a lot of references to Kepler's equation is a more accurate statement.
Title: Re: Problems with the Heliocentric Model
Post by: ChrisTP on August 07, 2019, 02:12:39 PM
Totallackey. If you think that because a planetarium floor is flat therefore the earth is flat then I could just stand on a football and claim that as the reason the earth is a sphere...

Anyway, I don't think a planetarium is really the same as an application in full 3D that you can effectively explore and visually see. For example if you look in said software and dr a line in 3D space from the exact position of a star down to the exact location on the planet earth that you are, that line would be coming in at the exact angle and direction as to where you'd visibly see the same star in real life (apart from maybe some slight degrees for refraction maybe). Can you do the same thing in a 3D application using a flat earth? Is there any such software? I would like to see it. regardless of the math involved, if the positions and angles all match up to real life for a 3D flat earth application that would be lovely. I think if there was such a software you'd see a hell of a lot of bending and warping, which you'd then have to somehow explain why those crazy bending lines make more sense than the almost perfectly straight lines from a globe earth.
Title: Re: Problems with the Heliocentric Model
Post by: robinofloxley on August 07, 2019, 02:57:04 PM
You introduced the software as evidence of the heliocentric model.

If it is, then the routines governing its operation, by default, are based on the laws and maths proposed by Kepler, Newton, and Einstein.

The reason you won't contact the developers of the software is simple.

You know the emperor is wearing no clothes.

If I understand you correctly you don't believe that planetarium software makes use of Kepler's equations. Well Starry Night is commercial software, so you can't easily look at the source code, however there is a very good open source alternative Stellarium and you can easily check the code for yourself.

https://github.com/Stellarium/stellarium/blob/master/plugins/Satellites/src/gsatellite/sgp4ext.cpp (https://github.com/Stellarium/stellarium/blob/master/plugins/Satellites/src/gsatellite/sgp4ext.cpp)

Take a look at line 159 onwards:

Quote
function newtonnu ... this function solves keplers equation...

Basically it's using Kepler's equation and a simplified perturbation model SGP4 https://en.wikipedia.org/wiki/Simplified_perturbations_models (https://en.wikipedia.org/wiki/Simplified_perturbations_models) for satellite calculations.

I can't speak for Starry Night, but certainly something in Stellarium utilizes Kepler.
Simply referring to Kepler or Newton does not in fact mean the program utilizes Kepler or Newton.

It certainly makes no reference to whether this particular section of code includes a resultant modeling output of all of the solar system, or even three body modeling.

So yes, while it makes mention, there is no real evidence those maths or equations are even utilized.
If I were sitting in a candlelit room with a quill pen and some trig tables trying to work out a planetary position, I'd no doubt be trying to use Kepler's equation, however these days we have powerful computers and we have far more sophisticated models which use numerical methods to deal with the perturbations inherent in the N-body problem.

For obvious reasons the JPL have a vested interest in celestial mechanics and they have produced a whole series of more and more refined models over time https://en.wikipedia.org/wiki/Jet_Propulsion_Laboratory_Development_Ephemeris (https://en.wikipedia.org/wiki/Jet_Propulsion_Laboratory_Development_Ephemeris).

So if I were trying to write my own planetarium software today and wanted really accurate results, my starting point would be VSOP87 https://en.wikipedia.org/wiki/VSOP_(planets) (https://en.wikipedia.org/wiki/VSOP_(planets)) which is based on JPL's DE200 ephemeris model. This is what Stellarium and a number of other similar projects use, I don't know about Starry Night.

Using VSOP87 is reasonably straightforward. Understanding it less so. http://neoprogrammics.com/vsop87/vsop87_theory_paper/VSOP87%20Theory%20Paper.pdf (http://neoprogrammics.com/vsop87/vsop87_theory_paper/VSOP87%20Theory%20Paper.pdf) however it certainly does include a lot of math!

The section of this paper explaining how the VSOP87 solution is derived states:

Quote
We must first solve Kepler's equation in order to get the expressions of the variables X, Y, Z.

So it seems that VSOP87 (upon which Stellarium's planetary positioning is based) does indeed make use of a lot of math and Kepler's equation.
It makes a use of a lot of references to Kepler's equation is a more accurate statement.

OK, you asked newhorizons for evidence of:

Quote
a rendered simulation of the solar system, based on the combined math of Kepler, Newton, and Einstein

I have provided you evidence that a rendered simulation of the solar system (Stellarium) does (according to the authors) at the very least use Kepler's equation and I've literally pointed you at the lines of source code involved. I have provided evidence that the authors of the definitive paper describing VSOP87 (which is the method Stellarium - and others - uses for planetary positioning) also claim to use Kepler's equation.

You did not ask for proof, you asked for evidence. I have provided you with the evidence you asked for, so now you shift the goalposts and you appear to be accusing the authors of the software and the paper describing VSOP87 of lying when they make their claims about using Kepler's equation.

Just think about this for a second. The authors in both cases did not need to mention Kepler at all. By claiming, in publicly available media, they were using Kepler's equations, both sets of authors were exposing themselves to scrutiny from anyone capable of reading the math and/or understanding code. Why would they do this if the claims were bogus?

I think it's pretty clear that you didn't expect anyone to actually go to the trouble of digging out the evidence you said you wanted, so you now have to backpedal and dismiss the evidence and essentially say "prove the authors aren't lying".

So, if in testing it is clearly demonstrated the inputs of these programs include nothing containing the math from Kepler, Newton, and Einstein, where does that leave you?

And where does this leave you now?
Title: Re: Problems with the Heliocentric Model
Post by: totallackey on August 07, 2019, 03:25:29 PM
Totallackey. If you think that because a planetarium floor is flat therefore the earth is flat then I could just stand on a football and claim that as the reason the earth is a sphere...
There is the issue concerning you see no sphericity from any vantage point while standing on flat ground.
Anyway, I don't think a planetarium is really the same as an application in full 3D that you can effectively explore and visually see.
Full 3D  is exactly what you see all around you, each and every day.
For example if you look in said software and dr a line in 3D space from the exact position of a star down to the exact location on the planet earth that you are, that line would be coming in at the exact angle and direction as to where you'd visibly see the same star in real life (apart from maybe some slight degrees for refraction maybe). Can you do the same thing in a 3D application using a flat earth?
As stated earlier and yet to be refuted...a planetarium.
Is there any such software? I would like to see it. regardless of the math involved, if the positions and angles all match up to real life for a 3D flat earth application that would be lovely. I think if there was such a software you'd see a hell of a lot of bending and warping, which you'd then have to somehow explain why those crazy bending lines make more sense than the almost perfectly straight lines from a globe earth.
If you are looking at software not clearly using Kepler, Newton, and Einstein, I think that, by default, qualifies as FE software.
Title: Re: Problems with the Heliocentric Model
Post by: totallackey on August 07, 2019, 03:33:15 PM
OK, you asked newhorizons for evidence of:

Quote
a rendered simulation of the solar system, based on the combined math of Kepler, Newton, and Einstein

I have provided you evidence that a rendered simulation of the solar system (Stellarium) does (according to the authors) at the very least use Kepler's equation and I've literally pointed you at the lines of source code involved.
No, you pointed me and other members to an overview of what the authors claim to contain such code.

Furthermore, as I have pointed out, the material you provided makes no claim or reference whether or not the lines of code or Kepler's equations are even utilized in performing the rendering or yielding a visual representation. 
I have provided evidence that the authors of the definitive paper describing VSOP87 (which is the method Stellarium - and others - uses for planetary positioning) also claim to use Kepler's equation.

You did not ask for proof, you asked for evidence. I have provided you with the evidence you asked for, so now you shift the goalposts and you appear to be accusing the authors of the software and the paper describing VSOP87 of lying when they make their claims about using Kepler's equation.
You are correct.

I asked for evidence.

I am not accusing anyone of lying, certainly not you or the programmers/developers of the software.

I have factually explained what was written in the source you provided and have shown what you have provided is evidence of mentioning Kepler and equations, not evidence it is utilized in rendering visuals.

Furthermore, when it comes VSOP87, I find this:

"The first step in computing the apparent position of a planet is computing the heliocentric coordinates (such as X,Y,Z) for both the planet and the Earth at the same moment."

So it appears that, even if this program is utilized, it does not render a 3 - body resolution to the rendering, nor can it render an entire solar system.
Just think about this for a second. The authors in both cases did not need to mention Kepler at all.
Quite true.

They did not.
By claiming, in publicly available media, they were using Kepler's equations, both sets of authors were exposing themselves to scrutiny from anyone capable of reading the math and/or understanding code. Why would they do this if the claims were bogus?
As I pointed out, there is nothing in writing indicating the use of Kepler's equation has anything to do with the visual outputs.
I think it's pretty clear that you didn't expect anyone to actually go to the trouble of digging out the evidence you said you wanted, so you now have to backpedal and dismiss the evidence and essentially say "prove the authors aren't lying".
I think it is actually quite clear, as explained here, I have done no such thing.
So, if in testing it is clearly demonstrated the inputs of these programs include nothing containing the math from Kepler, Newton, and Einstein, where does that leave you?

And where does this leave you now?
Where I was prior to your reply to this thread.
Title: Re: Problems with the Heliocentric Model
Post by: newhorizons on August 07, 2019, 04:16:59 PM
Quote
For example if you look in said software and dr a line in 3D space from the exact position of a star down to the exact location on the planet earth that you are, that line would be coming in at the exact angle and direction as to where you'd visibly see the same star in real life (apart from maybe some slight degrees for refraction maybe)

That is correct. If you were to place an astronaut floating freely in space somewhere they would see stars all around them. It would be as if they were located at the centre of some huge sphere and the stars were shining away from the inside surface of said sphere.  This is what astronomers call the celestial sphere although as I said before it is not really a sphere even if Totallackey thinks it is. 

It stands to reason then that our astronaut can see the whole of the celestial sphere from one spot.  They can look in whatever direction they want.  Now put the astronaut on a large spherical body also floating freely in space. He can walk freely all over the surface of the sphere and as he does so he will see different parts of the celestial sphere and hence different stars as the direction of his line of sight into space changes. This is exactly what happens on Earth.  We see different stars on the same night according to latitude and the rotation of the Earth and we see different stars seasonally due to the rotation of the Earth around the Sun.  Living in the northern hemisphere I never see the southern stars because my line of sight into space never allows for me to see them.

You simply cannot achieve this if you assert that the Earths surface is flat without introducing some serious levels of refraction that would distort beyond recognition the patterns of stars as we see them.
Title: Re: Problems with the Heliocentric Model
Post by: robinofloxley on August 07, 2019, 04:59:40 PM
OK, you asked newhorizons for evidence of:

Quote
a rendered simulation of the solar system, based on the combined math of Kepler, Newton, and Einstein

I have provided you evidence that a rendered simulation of the solar system (Stellarium) does (according to the authors) at the very least use Kepler's equation and I've literally pointed you at the lines of source code involved.
No, you pointed me and other members to an overview of what the authors claim to contain such code.

I provided a direct link to the C++ source code on github where the project is hosted. This is the actual source code used to build Stellarium. You can download this source code and build it yourself if you have the appropriate tools. I pointed you to a module called sgp4ext.cpp which contains a function called newtonnu, 248 lines of C++ source code which according to the author's comment "solves keplers equation". This is not an overview, it's actual source code. Whether the function does indeed solve Kepler's equation, I couldn't say, but this is clearly what the author claims.

Furthermore, as I have pointed out, the material you provided makes no claim or reference whether or not the lines of code or Kepler's equations are even utilized in performing the rendering or yielding a visual representation.

Well in an earlier post you said...

What makes him think it is simply pattern inputs is the fact if you publish the code, you won't find anything related to Kepler, Newton, or Einstein.

Now this is evidently published code and it does indeed contain something related to Kepler. If you now want proof that the code in question is used in the rendering, well that's moving the goalposts, this is not what you originally asked for. Stellarium contains about 700MB of source code. Tracing through that amount of someone else's unfamiliar code to determine if some return value is actually used for rendering would be a considerable undertaking and - call me cynical - but I think if I told you I'd done that and was satisfied, you would not take my word for it.

I have provided evidence that the authors of the definitive paper describing VSOP87 (which is the method Stellarium - and others - uses for planetary positioning) also claim to use Kepler's equation.

You did not ask for proof, you asked for evidence. I have provided you with the evidence you asked for, so now you shift the goalposts and you appear to be accusing the authors of the software and the paper describing VSOP87 of lying when they make their claims about using Kepler's equation.
You are correct.

I asked for evidence.

I am not accusing anyone of lying, certainly not you or the programmers/developers of the software.

I have factually explained what was written in the source you provided and have shown what you have provided is evidence of mentioning Kepler and equations, not evidence it is utilized in rendering visuals.

Furthermore, when it comes VSOP87, I find this:

"The first step in computing the apparent position of a planet is computing the heliocentric coordinates (such as X,Y,Z) for both the planet and the Earth at the same moment."

So it appears that, even if this program is utilized, it does not render a 3 - body resolution to the rendering, nor can it render an entire solar system.

Can I point you to page 311 of the paper where it says:

Quote
VSOP 87 contains the newtonian perturbations of the eight planets between themselves, the perturbations of the Moon on the Earth-Mood barycenter and on all the planets and the relativistic perturbations expressed in isotropic coordinates.

I think that speaks for itself.
Title: Re: Problems with the Heliocentric Model
Post by: newhorizons on August 07, 2019, 07:16:53 PM
I promised you an image of Jupiter and the compared prediction given by SN.  Please find attached. The shadow is highlighted by the red line and the Io satellite itself by the green line.  As you can see an almost perfect match. Lot of air currents rising still so Jupiter very fuzzy. Also still almost full daylight as Sun not quite set.
Title: Re: Problems with the Heliocentric Model
Post by: newhorizons on August 07, 2019, 07:21:27 PM
Larger scaled version of Jupiter image.
Title: Re: Problems with the Heliocentric Model
Post by: newhorizons on August 07, 2019, 07:28:46 PM
Just for good measure I increased the FOV of the camera slightly and increased the gain to reveal all four brightest satellites.  Still daylight so pleased I got them at all. I have highlighted with coloured lines the respective satellites which you can then identify on the SN screenshot.  Again a perfect match between real sky and simulation.  Being the largest of the satellites, and the largest natural satellite in the solar system, Ganymede shows the brightest image.
Title: Re: Problems with the Heliocentric Model
Post by: newhorizons on August 07, 2019, 07:42:01 PM
Finally for now, a nice close up of craters in the Moons southern hemisphere. 2000 frames stacked using Autostakkert and then sharpened with Registax.
Title: Re: Problems with the Heliocentric Model
Post by: totallackey on August 08, 2019, 10:53:59 AM
I provided a direct link to the C++ source code on github where the project is hosted. This is the actual source code used to build Stellarium. You can download this source code and build it yourself if you have the appropriate tools. I pointed you to a module called sgp4ext.cpp which contains a function called newtonnu, 248 lines of C++ source code which according to the author's comment "solves keplers equation". This is not an overview, it's actual source code. Whether the function does indeed solve Kepler's equation, I couldn't say, but this is clearly what the author claims.
Kepler's equation being solved...

How does that relate to the supposed heliocentricity of the solar system?
Furthermore, as I have pointed out, the material you provided makes no claim or reference whether or not the lines of code or Kepler's equations are even utilized in performing the rendering or yielding a visual representation.

Well in an earlier post you said...

What makes him think it is simply pattern inputs is the fact if you publish the code, you won't find anything related to Kepler, Newton, or Einstein.
Correct, I did write that, and in context of the discussion, leads one to ask the question that I asked above.

Repeating: How does that relate to the supposed heliocentricity of the system?
Now this is evidently published code and it does indeed contain something related to Kepler. If you now want proof that the code in question is used in the rendering, well that's moving the goalposts, this is not what you originally asked for.
Not moving the goalposts as the OP itself is entitled "Problems with the heliocentric model."

IF the source code does contain math and equations specifically by Kepler, we all know Kepler and his math and equations alone cannot explain a solar system that is heliocentric.

Very basic stuff.
contains about 700MB of source code. Tracing through that amount of someone else's unfamiliar code to determine if some return value is actually used for rendering would be a considerable undertaking and - call me cynical - but I think if I told you I'd done that and was satisfied, you would not take my word for it.
IF you showed your work and the work was legit, what difference would it make if I or anyone else failed to take your word for it?
Can I point you to page 311 of the paper where it says:

Quote
VSOP 87 contains the newtonian perturbations of the eight planets between themselves, the perturbations of the Moon on the Earth-Mood barycenter and on all the planets and the relativistic perturbations expressed in isotropic coordinates.

I think that speaks for itself.
Yes, it does...

As you point out (quite correctly, incidentally) this does conclusively state nothing in terms of modeling a heliocentric solar system.

More shockingly, let us consider the following: If these computer simulations you offer in defense of an heliocentric solar system are in fact a representation of the real deal, wouldn't that be considered SOLVING THE 3-BODY PROBLEM?

Can't the programmers of the software quickly run to ANY ESTEEMED UNIVERSITY or NASA and say, with 100 percent certainty, "HEY ALL!!! LOOKIE HERE!!! 3-BODY PROBLEM SOLVED!!!"?
Title: Re: Problems with the Heliocentric Model
Post by: newhorizons on August 08, 2019, 11:01:57 AM
Well I hope you liked the photos.

One thing I did notice in Stellarium yesterday is that it gives an absolute magnitude for Jupiter of -9.4.

The absolute magnitude is the brightness of a celestial body as seen from a standard distance of 10pc or 32.6 light years. Since the Sun has an absolute magnitude (correctly given by Stellarium) of +4.8 I am not sure where the -9.4 comes from.

Anyway I will leave you to argue between yourselves regarding the Stellarium source code for now.
Title: Re: Problems with the Heliocentric Model
Post by: totallackey on August 08, 2019, 11:37:01 AM
Well I hope you liked the photos.

One thing I did notice in Stellarium yesterday is that it gives an absolute magnitude for Jupiter of -9.4.

The absolute magnitude is the brightness of a celestial body as seen from a standard distance of 10pc or 32.6 light years. Since the Sun has an absolute magnitude (correctly given by Stellarium) of +4.8 I am not sure where the -9.4 comes from.

Anyway I will leave you to argue between yourselves regarding the Stellarium source code for now.
Those photos are terrific!

You do excellent work!
Title: Re: Problems with the Heliocentric Model
Post by: newhorizons on August 08, 2019, 12:23:29 PM
Well thank you. I can produce color images of Jupiter by using separate red, green and blue filters.

Fine details such as a very small feature on Jupiters disk are best recorded using a red filter as longer wavelengths of light are less affected by turbulence. I plan to get a 683nm filter which is a very deep red.

I have been imaging for sone years now so results are getting better. If there is anythng else you would like me to image for you (rings of Saturn is a popular request) then just let me know.
Title: Re: Problems with the Heliocentric Model
Post by: robinofloxley on August 08, 2019, 01:29:10 PM
Kepler's equation being solved...

How does that relate to the supposed heliocentricity of the system?

What makes him think it is simply pattern inputs is the fact if you publish the code, you won't find anything related to Kepler, Newton, or Einstein.
Correct, I did write that, and in context of the discussion, leads one to ask the question that I asked above.

Repeating: How does that relate to the supposed heliocentricity of the system?

Not moving the goalposts as the OP itself is entitled "Problems with the heliocentric model."

IF the source code does contain math and equations specifically by Kepler, we all know Kepler and his math and equations alone cannot explain a solar system that is heliocentric.

Very basic stuff.

I think that speaks for itself.
Yes, it does...

As you point out (quite correctly, incidentally) this does conclusively state nothing in terms of modeling a heliocentric solar system.

Thank you.

I appreciate that the OP and the thread title are concerned with "Problems with the Heliocentric Model", however you've made various specific claims and challenges in regard to modelling software.

In particular, you asked newhorizons to "confirm the the product you offer utilizes the math utilized by Kepler, Newton, and Einstein" and then went on to claim "if you publish the code, you won't find anything related to Kepler, Newton, or Einstein".

Stellarium is open source which means all of the source is published, so I looked at the code myself and found code related to Kepler, directly addressing your claim.

You threw out a challenge saying "show me the code!" So I did.

How does this relate to heliocentricity? It doesn't. I've never mentioned heliocentricity. Our exchanges so far on this topic have all been about whether certain types of software does or does not contain code related to Kepler. Can we at least agree that I have shown you some code which meets the criteria of your original challenge? I accept that you do now want to qualify your original challenge and establish that the code is actually used in the rendering and I agree that's a perfectly valid question to ask, but it was not part of the original challenge and I'm not prepared to spend the time and effort on trying to figure that one out, so it'll have to remain an unanswered question.

You've also said "we all know Kepler and his math and equations alone cannot explain a solar system that is heliocentric". Well it depends what you mean. You can build a model based on Kepler's ideas alone and it'll give you reasonably accurate results, but time has moved on and better models have emerged (and continue to emerge) which produce ever more accurate results. If you want to build a planetarium model, or plan a mission to Titan you'd be silly not to use a more sophisticated model which handles the N-body problem and the perturbations which are introduced.
Title: Re: Problems with the Heliocentric Model
Post by: robinofloxley on August 08, 2019, 01:36:26 PM
Well I hope you liked the photos.

One thing I did notice in Stellarium yesterday is that it gives an absolute magnitude for Jupiter of -9.4.

The absolute magnitude is the brightness of a celestial body as seen from a standard distance of 10pc or 32.6 light years. Since the Sun has an absolute magnitude (correctly given by Stellarium) of +4.8 I am not sure where the -9.4 comes from.

Anyway I will leave you to argue between yourselves regarding the Stellarium source code for now.
Those photos are terrific!

You do excellent work!

Well there's one thing at least we completely agree upon  :) - great photos!
Title: Re: Problems with the Heliocentric Model
Post by: totallackey on August 09, 2019, 10:30:54 AM
I appreciate that the OP and the thread title are concerned with "Problems with the Heliocentric Model", however you've made various specific claims and challenges in regard to modelling software.

In particular, you asked newhorizons to "confirm the the product you offer utilizes the math utilized by Kepler, Newton, and Einstein" and then went on to claim "if you publish the code, you won't find anything related to Kepler, Newton, or Einstein".
Yeah, any honest reading of what I wrote would understand the inclusion of MODELING the results of the math of Kepler, Newton, and Einstein, in visual form.

If, by chance, you didn't understand that when reading my written point, then please understand that point at this time.
Stellarium is open source which means all of the source is published, so I looked at the code myself and found code related to Kepler, directly addressing your claim.

You threw out a challenge saying "show me the code!" So I did.
And, as pointed out, what you referred to indicates Kepler's equation is solved, according to the overview.

There is only a statement it has been solved, and if it is contained in that software program, it would still have nothing to do with the rendering of visual outputs within Stellarium (or more precisely, a full visual rendering of the solar system).
How does this relate to heliocentricity? It doesn't. I've never mentioned heliocentricity. Our exchanges so far on this topic have all been about whether certain types of software does or does not contain code related to Kepler. Can we at least agree that I have shown you some code which meets the criteria of your original challenge?
We can agree you referred the members to an overview containing writing that Kepler's equation is solved.
I accept that you do now want to qualify your original challenge and establish that the code is actually used in the rendering and I agree that's a perfectly valid question to ask, but it was not part of the original challenge and I'm not prepared to spend the time and effort on trying to figure that one out, so it'll have to remain an unanswered question.
Okay.

Thank you.
You've also said "we all know Kepler and his math and equations alone cannot explain a solar system that is heliocentric". Well it depends what you mean. You can build a model based on Kepler's ideas alone and it'll give you reasonably accurate results, but time has moved on and better models have emerged (and continue to emerge) which produce ever more accurate results. If you want to build a planetarium model, or plan a mission to Titan you'd be silly not to use a more sophisticated model which handles the N-body problem and the perturbations which are introduced.
Well, any rendering would seem to have solved the three-body problem, correct?

If so, why no solution announced from on high?

Why no smoke from the chimney?
Title: Re: Problems with the Heliocentric Model
Post by: robinofloxley on August 09, 2019, 11:48:20 AM
I appreciate that the OP and the thread title are concerned with "Problems with the Heliocentric Model", however you've made various specific claims and challenges in regard to modelling software.

In particular, you asked newhorizons to "confirm the the product you offer utilizes the math utilized by Kepler, Newton, and Einstein" and then went on to claim "if you publish the code, you won't find anything related to Kepler, Newton, or Einstein".
Yeah, any honest reading of what I wrote would understand the inclusion of MODELING the results of the math of Kepler, Newton, and Einstein, in visual form.

If, by chance, you didn't understand that when reading my written point, then please understand that point at this time.

My "honest reading" of what you wrote led me to think you were confident there wouldn't be any code related to Kepler et al in the code and I went to the trouble of downloading and searching the codebase because I thought that's what you meant. Sorry.

Stellarium is open source which means all of the source is published, so I looked at the code myself and found code related to Kepler, directly addressing your claim.

You threw out a challenge saying "show me the code!" So I did.
And, as pointed out, what you referred to indicates Kepler's equation is solved, according to the overview.

There is only a statement it has been solved, and if it is contained in that software program, it would still have nothing to do with the rendering of visual outputs within Stellarium (or more precisely, a full visual rendering of the solar system).

Well now you are asserting that it would have nothing to do with the rendering, but that's pure speculation and common sense says that if the author has taken the trouble to write code to solve Kepler's equation, add it to the codebase and keep it there over time, then they must have had some reason for doing so and given the purpose of Stellarium is to render images, it makes more sense to me that this code is used in the rendering. And now I'm speculating too. In reality neither of us knows for sure at this point.

How does this relate to heliocentricity? It doesn't. I've never mentioned heliocentricity. Our exchanges so far on this topic have all been about whether certain types of software does or does not contain code related to Kepler. Can we at least agree that I have shown you some code which meets the criteria of your original challenge?
We can agree you referred the members to an overview containing writing that Kepler's equation is solved.
I accept that you do now want to qualify your original challenge and establish that the code is actually used in the rendering and I agree that's a perfectly valid question to ask, but it was not part of the original challenge and I'm not prepared to spend the time and effort on trying to figure that one out, so it'll have to remain an unanswered question.
Okay.

Thank you.
You've also said "we all know Kepler and his math and equations alone cannot explain a solar system that is heliocentric". Well it depends what you mean. You can build a model based on Kepler's ideas alone and it'll give you reasonably accurate results, but time has moved on and better models have emerged (and continue to emerge) which produce ever more accurate results. If you want to build a planetarium model, or plan a mission to Titan you'd be silly not to use a more sophisticated model which handles the N-body problem and the perturbations which are introduced.
Well, any rendering would seem to have solved the three-body problem, correct?

If so, why no solution announced from on high?

Why no smoke from the chimney?

Announcement of what exactly? There's no new discovery being claimed here.

There are no analytic solutions for the N-body problem and analytic solutions for the more restricted 3-body solution only exist for limited special cases. But there are numerical solutions dating back decades and these are used for planetarium software and mission planning. Numerical solutions are only approximations, but the most sophisticated ones are highly accurate and in very close agreement with observation - they are still solutions. Stellarium and other planetarium software claim to use VSOP 87 as the basis for determining planetary positions. VSOP 87 is described by Wikipedia as "semi-analytic" - being a mixture of analytical and numerical methods. The paper explaining VSOP 87 says "We must first solve Kepler's equation in order to get the expressions of the variables X, Y, Z" - that would belong to the analytic side of semi-analytic.

My belief is this:
No attempt to claim proof of anything here, just laying out my view.
Title: Re: Problems with the Heliocentric Model
Post by: Tim Alphabeaver on August 09, 2019, 10:35:01 PM
If so, why no solution announced from on high?

Why no smoke from the chimney?
Numerical solutions and analytical solutions are not the same thing.
Either you already knew this and you're just trolling around, or you didn't know this and you should really go back to basics on this.
Title: Re: Problems with the Heliocentric Model
Post by: totallackey on August 10, 2019, 12:50:43 PM
If so, why no solution announced from on high?

Why no smoke from the chimney?
Numerical solutions and analytical solutions are not the same thing.
Either you already knew this and you're just trolling around, or you didn't know this and you should really go back to basics on this.
Ok.

Please explain the difference between an analytic solution and a numerical solution and how they are differentiated when it comes to a problem involving math?
Title: Re: Problems with the Heliocentric Model
Post by: Tim Alphabeaver on August 10, 2019, 04:03:51 PM
Please explain the difference between an analytic solution and a numerical solution and how they are differentiated when it comes to a problem involving math?
This post sums it up nicely:
https://www.quora.com/What-is-the-difference-between-a-numerical-and-an-analytical-solution (https://www.quora.com/What-is-the-difference-between-a-numerical-and-an-analytical-solution)

An analytical solution is exact.
A numerical solution is an approximation that's used when an analytical solution is unavailable or unusable.
Maybe you remember using the trapesium rule to estimate the area under a curve in maths class - that's using a numerical method instead of an analytic one. If the curve was simple enough you could have just used calculus to integrate it get an exact solution and you wouldn't need to bother with the trapesium rule.

If you're doing this for a living you'd use more complicated algorithms than the trapesium rule, and you'd generally know how accurate your numerical solution is.

This is why we can say that not having an analytical solution to the three-body problem is okay. You compute a numerical solution and know how good your solution is. This means that if I want to know the position of Jupiter in X years with an accuracy of Y meters, I can do that.

Title: Re: Problems with the Heliocentric Model
Post by: Tom Bishop on August 10, 2019, 04:46:51 PM
Sort of like of Ptolmy used numerical computations and epicycles to predict the location of the planets?
Title: Re: Problems with the Heliocentric Model
Post by: newhorizons on August 10, 2019, 08:56:59 PM
Tom,

As an amateur astronomer of many years experience I am used to watching and studying the night sky and would describe myself as having a better than average knowledge of how the planets 'behave' over the course of a year. Comparing what I am used to seeing in the real sky with the diagrams depicting the early Copernican heliocentric system and the Ptolemy geocentric system (both of which contain these epicycles for individual planets) I cannot say I agree with either diagram.

Some questions immediately come to mind. Firstly in relation to the epicycles why would the planets individually orbit around apparently nothing?  Why do the Moon and Sun not follow an epicycle? From a physics point of view they are just massive bodies like the planets so why should they be any different from the planets?

Finally if the planets are moving in circles (epicylces) in addition to their orbital paths then they would appear to 'wobble' w.r.t the background stars in a way which they don't. I suppose the epicycle could be some form of crude attempt to explain retrograde motion. If this is the case then it doesn't work.
Title: Re: Problems with the Heliocentric Model
Post by: Tim Alphabeaver on August 10, 2019, 09:42:36 PM
Sort of like of Ptolmy used numerical computations and epicycles to predict the location of the planets?
I'm not well-versed on Ptolemy's model, but it seems to me that it's simple enough model where each body moves around a fixed circle. In this case, each body's motion could be solved analytically, so no need for a numerical solution.
Title: Re: Problems with the Heliocentric Model
Post by: Tom Bishop on August 10, 2019, 11:15:57 PM
It says here that he was using numerical computations:

https://books.google.com/books?id=JVhTtVA2zr8C&pg=PA29&source=gbs_toc_r&cad=4#v=onepage&q&f=false

(https://i.imgur.com/tLawyDV.png)
Title: Re: Problems with the Heliocentric Model
Post by: Tim Alphabeaver on August 11, 2019, 11:18:07 AM
It says here that he was using numerical computations:

https://books.google.com/books?id=JVhTtVA2zr8C&pg=PA29&source=gbs_toc_r&cad=4#v=onepage&q&f=false
Then maybe they did use numerical solutions. Like I said - not an expert. I'd need a look at the equations to provide any useful input.
Title: Re: Problems with the Heliocentric Model
Post by: Tim Alphabeaver on August 11, 2019, 11:30:56 AM
Another thing to think about: I grabbed this from stack exchange:

In mathematics, an expression is said to be a closed-form expression if it can be expressed analytically in terms of a finite number of certain "well-known" functions.

From what I've read analytic solutions and closed-form solutions are the same thing.
I could imagine a case like y=x^2, which is definitely closed-form, to not be closed-form to someone like Ptolemy. How do you calculate a square (or sqrt(x), or sin(x)) in 100AD? The answer seems to be that you get a table that someone else wrote and look it up.
Title: Re: Problems with the Heliocentric Model
Post by: AATW on August 12, 2019, 10:17:33 AM
Sort of like of Ptolmy used numerical computations and epicycles to predict the location of the planets?
Actually yes, exactly like that.
But that model was later replaced with a better, heliocentric one. That's how science works.
You seem to struggle with this concept.
And Ptolmy wasn't a flat earther...
Title: Re: Problems with the Heliocentric Model
Post by: newhorizons on August 12, 2019, 09:52:42 PM
I have always enjoyed the Khan Academy.  Here is a good link that show some simulated demonstrations of solar system events.  A number of links to the left side to check out.

https://www.khanacademy.org/partner-content/nasa/measuringuniverse/spacemath1/a/the-geocentric-universe
Title: Re: Problems with the Heliocentric Model
Post by: robinofloxley on August 13, 2019, 09:13:24 AM
Sort of like of Ptolmy used numerical computations and epicycles to predict the location of the planets?
Actually yes, exactly like that.
But that model was later replaced with a better, heliocentric one. That's how science works.
You seem to struggle with this concept.
And Ptolmy wasn't a flat earther...

I've noticed Tom and others bring up epicycles more than once, but I don't really see the significance. I don't think Tom actually believes planetary orbits are physically epicycles, just that epicycles are a mathematical tool to predict orbits. OK, so epicycles of sufficient complexity could be used to predict orbits. So what. We have other tools now and we use them instead, so presumably these new tools are either more accurate or easier to use or produce results faster.
Title: Re: Problems with the Heliocentric Model
Post by: newhorizons on August 15, 2019, 09:51:09 PM
Regarding the 'problem' with the heliocentric model, doesn't the existence of stellar parallax kind of demonstrate that the Earth is orbiting the Sun?  After all if it wasn't then the stars would show no parallax would they.
Title: Re: Problems with the Heliocentric Model
Post by: Tom Bishop on August 18, 2019, 02:28:47 PM
Regarding the 'problem' with the heliocentric model, doesn't the existence of stellar parallax kind of demonstrate that the Earth is orbiting the Sun?  After all if it wasn't then the stars would show no parallax would they.

Some stars exhibit zero parallax, while other stars exhibit positive or negative parallax of about equal distribution. Stars which exhibit ''negative parallax'' travel in a direction contradictory to heliocentrism, and are usually dismissed as "errors".

http://web.archive.org/web/20100826022827/http://www.realityreviewed.com/Negative%20parallax.htm

Quote
A careful examination of photographic plates that have been exposed to the same region of sky, but at times that are a few months apart, will reveal the fact that some stars have shifted their position with respect to the 'background' stars. Such stars are assumed to be closer to us than the (effectively) infinitely far away 'background' stars, and the effect is naturally given the name of stellar parallax.

~

'''Negative Parallax'''

There are 1,058,332 objects in the Tycho Main Catalogue, and these have a median astrometric precision of 7 mas for visual magnitude 9 and below, increasing through 25 mas for visual magnitude 10-11.

Using the ESA's parameter entry table [5], we selected field three (parallax) and specified a range of -919 (min) to -20 (max) mas, over the entire dataset. This produced 262,100 records of negative parallax objects, or 25% of the total.

Next we selected the positive parallax objects via a minimum value of 20 mas and a maximum of 701.5 mas. This resulted in 310,758 records, or 29% of the total.

The remaining 46% of the Tycho Main Catalogue entries can be assumed to possess zero parallax, within the precision of (0 ± 20) mas.

Section 2.2 Contents of the Tycho Catalogue [6] makes the following statement regarding Field T11, "The trigonometric parallax, π, is expressed in units of milliarcsec. The estimated parallax is given for every star, even if it appears to be insignificant or negative (which may arise when the true parallax is smaller than its error)."

A further test was conducted, to see if the stars moving across the astrometric instrument slit were directionally different in the northern celestial hemisphere to what they were in the southern celestial hemisphere. In this case, as well as the parallax field, the declination field was also selected. Of the non-zero-parallax stars in the northern celestial hemisphere (0°N ≤ δ ≤ 90°N), 45% of them had a negative parallax, and in the southern celestial hemisphere (0°S ≤ δ ≤ 90°S), 46% of non-zero objects had a negative parallax. So here again is a very symmetrical distribution that would be typical of a naturally occurring phenomenon.

~

In Fig. 3, 46% of all stars are located between the limits indicated by the two dotted lines on either side of the mean (the centre point of the stellatum thickness), and from Fig. 2 we see that this would imply 27% of stellatum stars would be closer to us (and thus display positive parallax) and 27% would be further away than the majority (and thus display negative parallax). I.e., 46% are middle stars (as termed in Fig. 2), 27% are inner stars (c.f. 29% from the Tycho Main Catalogue), and 27% are outer stars (c.f. 25% from the Tycho Main Catalogue).

~
 
'''Conclusion'''

It is an indisputable fact that stellar parallax, like the phases of Venus, has been widely cited as 'proof' that the World orbits the Sun. This is unfortunate, since the phenomenon proves no such thing. The only thing it does prove is that either the World is moving with respect to the stars, or that the stars are moving with respect to the World.

At this the geocentrists usually rest their case, claiming that the adoption of a heliocentric philosophy is just as much a matter of faith as the adoption of a geocentric philosophy. However, this invocation of faith is unnecessary and unjustified, for if it were such a simple choice between the World going around the Sun, or some stars moving slightly in order to conveniently give the appearance of the World going around the Sun, then the heliocentrists would have a point of strong probability (as opposed to a point of proof) in their favour, and geocentrism would indeed become more faith than science. Contrariwise it is worthwhile noting that credibility as regards the sizes of the Sun and Moon discs producing the observed solar eclipse effect that we marvel at sits more comfortably with the intelligent design position that geocentrism tends to imply, rather than with the heliocentrists and their claim of coincidence.

The phenomenon of stellar parallax is not what we have been generally led to believe, because in exactly the same way that Eddington 'proved' Einstein's General Theory of Relativity in 1919 by rejecting, omitting or deleting 60% of his measurement data on the bending of starlight, so modern astrophysics maintains the misconception that parallax 'proves' the Kopernikan philosophy of the World hurtling around the Sun, by ignoring and dismissing the entire dataset of negative parallax measurements.

The ESA, unlike Eddington before them, have kept and filed data values which do not fit in with the ruling model of the universe, and should be commended for so doing, but nevertheless they do seem to dismiss a significant proportion of their measurements rather glibly. Of course, they do say that these may arise due to measurement error, but the number and symmetrical distribution of these values would tend to deny this as being anything other than an exception to the rule.

Furthermore, although angular parallax measurements are small (the largest positive value gives an angle ACB, in Fig. 1, on the order of only 0.7 of an arcsecond), the effect is known to be genuine by way of photographic plates taken at various times over a period of twelve months which clearly show the same slight movement of some stars with respect to the background star field. In other words, stellar parallax is an observable phenomenon that is repeatable, rather than being experimental or statistical errors in measurement.

When the full picture is revealed and considered, therefore, it is clearly geocentrism that has the potential to fully and adequately account for the hundreds of thousands of negative parallax observations that have now been recorded, although it is acknowledged that a detailed explanation is not currently available.

Author information: https://web.archive.org/web/20031113155858/http://www.erionline.co.uk/Profile%20Dr%20Neville%20Jones.htm

Title: Re: Problems with the Heliocentric Model
Post by: newhorizons on August 18, 2019, 02:43:24 PM
https://physics.stackexchange.com/questions/244645/negative-parallax

https://astronomy.stackexchange.com/questions/26250/what-is-the-proper-interpretation-of-a-negative-parallax

http://adsabs.harvard.edu/full/1943AnDea...4....1L

http://www.mpia.de/~calj/parallax.pdf

Negative parallax can be more of a statistical result rather than an actual measurement. And therefore not evidence against heliocentricism.
Title: Re: Problems with the Heliocentric Model
Post by: Tom Bishop on August 18, 2019, 03:04:30 PM
https://physics.stackexchange.com/questions/244645/negative-parallax

https://astronomy.stackexchange.com/questions/26250/what-is-the-proper-interpretation-of-a-negative-parallax

http://adsabs.harvard.edu/full/1943AnDea...4....1L

http://www.mpia.de/~calj/parallax.pdf

Negative parallax can be more of a statistical result rather than an actual measurement. And therefore not evidence against heliocentricism.

According to the author that I quoted the star catalog shows that there were about as many stars in negative parallax as there are positive parallax. Positive = 29% of the total. Negative = 25% of total. Remainder had zero parallax. If true, then there are thousands of stars exhibiting negative parallax. The ones which support your model are correct, and the ones which contradict your model are wrong and errors? Very interesting.
Title: Re: Problems with the Heliocentric Model
Post by: AATW on August 18, 2019, 03:22:34 PM
The ones which support your model are correct, and the ones which contradict your model are wrong and errors? Very interesting.
Wow. I mean, you’re right that one shouldn’t cherry pick but that is literally all you do so it’s a bit rich you raising your eyebrow at it.

From what I understand (admittedly I’m no expert in this field) the parallax for distant stars should be so close to zero that the error inherent in taking these measurements means you’ll sometimes get a negative and sometimes get a positive. Actually neither are right. For closer objects the amount of parallax is big enough compared with the error that the measurements can be used to estimate the distance to them
Title: Re: Problems with the Heliocentric Model
Post by: newhorizons on August 18, 2019, 03:39:38 PM
Quote
If true, then there are thousands of stars exhibiting negative parallax

Only thousands Tom..  do you know how many stars we estimate there are in the Milky Way Galaxy alone?  I know single star clusters alone which contain a million stars or more so 'thousands' is a very small fraction indeed and given the very small angles involved then it is hardly surprising.  I would have thought the number would be more like tens of thousands. Even that would be a very small fraction.

Parallax though is just one of numerous points of evidence that support the heliocentric model.  Remind me again... what is meant by Occams Razor?

Title: Re: Problems with the Heliocentric Model
Post by: Tom Bishop on August 18, 2019, 04:07:58 PM
Well, it said hundreds of thousands:

Quote
There are 1,058,332 objects in the Tycho Main Catalogue, and these have a median astrometric precision of 7 mas for visual magnitude 9 and below, increasing through 25 mas for visual magnitude 10-11.

Using the ESA's parameter entry table [5], we selected field three (parallax) and specified a range of -919 (min) to -20 (max) mas, over the entire dataset. This produced 262,100 records of negative parallax objects, or 25% of the total.

Next we selected the positive parallax objects via a minimum value of 20 mas and a maximum of 701.5 mas. This resulted in 310,758 records, or 29% of the total.

The remaining 46% of the Tycho Main Catalogue entries can be assumed to possess zero parallax, within the precision of (0 ± 20) mas.

The history of this is interesting...

https://books.google.com/books?id=b2l8cO4wpL4C&pg=PA63&dq=%22a+large+negative+parallax+of%22&hl=en&sa=X&ved=0ahUKEwjzguuE49LfAhUQQq0KHTO_CCYQ6AEIKjAA#v=onepage&q&f=true


(https://i.imgur.com/EZlzxJc.png)
(https://i.imgur.com/4pWl6AJ.png)

The above astronomer says that astronomers tend to jump to subjective instrumental errors at the drop of hat. He points out and declares that y Draconis has a large negative parallax and aberration that is impossible to accept, and that this was found and verified by others, by "however and whomever treated the outcome".
Title: Re: Problems with the Heliocentric Model
Post by: newhorizons on August 18, 2019, 04:46:12 PM
Hundreds of thousands is still a very small number when it comes to stellar populations.

The GAIA satellite is set to measure around a billion stars and that is still less than 1% of the estimated population of the Milky Way.
Title: Re: Problems with the Heliocentric Model
Post by: garygreen on August 18, 2019, 07:05:34 PM
noisy data can easily produce negative parallax values:
(https://i.imgur.com/Yb7ibMF.png)

it's relatively simple to demonstrate that negative parallax values are the result of measurement error.  you simulate a set of known parallax values, and you recover them with mock measurements.  even if you give your mock measurements only gaussian noise (no systematics), you will still recover negative parallax values.

Luri+2018 demonstrates this here: https://github.com/agabrown/astrometry-inference-tutorials/blob/master/luminosity-calibration/DemoNegativeParallax.ipynb

for an in-depth treatment of the these measurement uncertainties as they pertain to gaia, see:

https://arxiv.org/pdf/1804.09376.pdf
https://arxiv.org/pdf/1507.02105.pdf
https://arxiv.org/pdf/1804.10121.pdf
https://github.com/agabrown/astrometry-inference-tutorials/

the punchline is that parallax catalogs are dominated by faint, distant sources (this is true of all astronomical surveys).  faint, distant sources will have small parallaxes (they are distant), and large positional measurement uncertainties (they are faint).
Title: Re: Problems with the Heliocentric Model
Post by: newhorizons on August 18, 2019, 07:19:54 PM
Your history link Tom makes for interesting reading.  From a quick glance I would say that particular document dates to about the mid 19th century. Given the choice between what that says and the data obtained from the GAIA satellite I think I know which option I would choose.  How about you?

What garygreen says makes complete sense and provides a very clear explanation as to why these errors that produce apparent -ve parallaxes occur.
Title: Re: Problems with the Heliocentric Model
Post by: Tim Alphabeaver on August 18, 2019, 09:20:52 PM
If true, then there are thousands of stars exhibiting negative parallax.
I just looked this up myself in the 'Hipparcos, the New Reduction' catalogue. There are 23,000 results when I filter parallax >10 mas, and only 168 results when I filter for parallax of <-10 mas. This number goes down to 50 results (0.04%) when I filter <-20 mas, which was the test used in your quoted text before.

It looks like this problem goes away when you look at more recent, accurate catalogues. This is consistent with it being caused by uncertainty in observations.
Title: Re: Problems with the Heliocentric Model
Post by: newhorizons on August 18, 2019, 09:39:55 PM
Tom,

You state in your earlier post that an astronomer in the article that your link to (mid 19th century?) mentions that the star Gamma (Y) Daconis shows a large negative parallax.  There are several other descriptions of this star (assuming that is the star you mean, also known as Eltanin) which seem to offer a different view on this.  For example...

https://www.universeguide.com/star/eltanin

It actually quotes two different parallax measurements spaced 10 years apart.  The figures are given towards the end of the page.  You will appreciate that an angle of just a few milli (thousandths of) arc seconds is very small and would be extremely difficult to measure with the kind of equipment available to an astronomer in the mid 1850s so it is no wonder he struggled.


Just noticed... I think the article in your link was actually published in 1902.  Makes no difference though... to my case stated above.

Related to the topic of stellar parallax, I note it states in FE Wiki,

Quote
Firstly, we must understand that the stars in FE are small and a few thousand miles above the sea level of the earth. This change in distance compared to RE figures is due to an adjusted astronomical parallax on a Flat Earth. The angle of stellar parallax changes, as it does with the sun, when the earth is assumed to be a flat surface.

We should actually understand that the stars are neither small or just a few thousand miles away. They might need to be for FE theory to work but in reality they are not and astronomers have got good evidence to show that they are not.   Secondly... solar parallax?  How does that work since we don't need parallax to work out the distance of the Sun?
Title: Re: Problems with the Heliocentric Model
Post by: somerled on August 19, 2019, 08:33:20 AM
The author of that linked article does state that the calculations of proper motion , and others (all items in red ) , could be incorrect so why should they be accepted as correct ?
Title: Re: Problems with the Heliocentric Model
Post by: newhorizons on August 19, 2019, 08:58:44 AM
Are the quoted figures for the parallax measurements shown in red?

I think you will find the only estimated or calculated values are those for radius and temperature.
Title: Re: Problems with the Heliocentric Model
Post by: somerled on August 19, 2019, 09:53:25 AM
My mistake - I read that as saying " The items in red "  and the paragraphs headings for  radius , temp , radial velocity and proper motion are all in red .
Title: Re: Problems with the Heliocentric Model
Post by: newhorizons on August 19, 2019, 11:02:53 AM
Agreed the author should have used a different color for the text labelling sub-headings but the descriptions before where it says 'visual facts' explains which values are calculated or estimated. Those for parallax are not among them.

Its amazing to think satellites can now measure parallax angles down to a few milliarc seconds. GAIA is set to produce the most detailed ever 3D map of the Milky Way Galaxy. A friend of mine works at E2V, the company involved in designing and building the cameras on board GAIA. He was in the team that lab tested the prototype chips. Biggest chip array I have ever seen!